Anda di halaman 1dari 100

B

iology is the study of life and teaches us about ourselves and the natural world around us. A good
starting point when studying Biology is, to admire the perfection of nature and the principles of life.
Many people find Biology the hardest and the most boring subject at school. Thats not true, indeed, if
you know how to appreciate the beauty of Biology then you can get good results in this subject.
Studying for Biology classes is very different from studying for history or English classes. Strategies that
worked well in those classes may not work well here. Biology classes demand that you master difficult
concepts such as DNA replication, evolution and the citric acid cycle. You must also have command on
biological terms like meiosis, alleles, homeostasis, monocots and eukaryotes. That means Biology demands
recall of specific concepts and terminology. Youre not going to learn this subject in one night before the
exam, so you need to begin with a realistic study strategy. You can try following tips to improve your
performance in biology.

READ WITH A DIFFERENCE


When you are assigned reading from the textbook, you need to do much more than run your eyes over the pages. The most successful
students read with pencil in hand and they actively engage the ideas presented in the book. Every time you are assigned reading,
write down essential information in a notebook. Your reading notes will help you process difficult material and retain what you read.
They will also serve as a study guide as you prepare for the exam. As you read, record the following in your notebook
l
Key terms and their definitions. Dont simply pass over unfamiliar terms. Look them up and write them down. Biology demands
a lot of memorisation.
l
Summaries of central concepts. If youre studying the human digestive system, write in your own words a summary of the
digestive process. If youre unable to do so, you havent yet mastered the concept.
l
Important diagrams. Even if you are nt an artist, the act of drawing cellular structures or ecological systems will help you
remember the terminology and underlying concepts.

MEMORISE WITH A DIFFERENCE


One of the great challenges of Biology is the large number of new terms and concepts you need to memorise. This is especially
difficult if you have, like many students, a memory like a sieve. But dont despairthere are many ways to improve your retention of
new material:
l
Flash cards Write terms and concepts on the front of 3 5 cards and definitions and descriptions on the back. Use these to test
yourself. Better yet, get together with a friend and test each other.
l
l
Use the terminology Make up sentences that employ one picture relates to another and so on, until you can pull
difficult and unfamiliar words. Using specialised terms will the whole lecture or chapter together.
help you learn them much more effectively than simply l
Do you create your own flow charts or concept maps to
reading them. organise large quantities of information? Get a set of
l
Teach the material With a friend, find an empty classroom coloured pencils or pens and get creative! A Learning
and teach the material to each other. Speaking and writing on Instructor can help you practice this technique, using your
the board will quickly move information to your long-term own class notes and textbook.
memory. l
Do you study with a small group of dedicated students?
l
Develop mnemonics Come up with catchy phrases to help It is helpful to get different perspectives. Have each
you remember sequences and terms. For example, Must Be member bring five practice questions to each study session
Good can help you remember stages of embryo to quiz each other. Try to write questions in the same
development: Morula, Blastula, Gastrula. format(s) that your professor uses. If you dont feel
comfortable with an answer, take your sample problem and
MORE AND MORE PRACTICE MAKES YOU answer to the professor, discussion group leader, or TA.
PERFECT They will be impressed by your efforts to master the topic!
l
Do you attend review sessions? Bring specific,
Throughout the semester, keep asking and answering questions
well-thought out questions about topics or areas you are
related to what you are learning. Memorisation is only part of the
still trying to master. Ask a member of your study group to
equation. You need to understand what you study. join you; compare notes afterward.
l
Test yourself with the textbook Chances are each chapter in l
Do you review the various topics or practice questions
your textbook concludes with practice questions. Use these to
in random order? This is important to do when studying
test your understanding of the material.
so that your mind can become accustomed to jumping
l
Turn your notes into questions For each lecture and from topic to topic on the test. Old exams are useful for this
reading assignment, figure out what are the key concepts and purpose.
turn them into potential exam questions. Practice writing l
Do you analyse your previous tests? This is a powerful
answers to the questions.
study tool. Examine each question. Look at what you got
l
Talk Biology with classmates Youll master biology quickly right and ask yourself why you got it right. Do the same for
if you become conversant in the concepts and terminology. the questions you got wrong. Make a thoughtful and well
Try getting together with classmates on a regular basis to considered comparison between your answers and the
discuss course content. answers in the key and ask yourself what you would have
l
Make it real Since Biology is the study of the living world needed to do differently to get the question right.
around you, dont limit your study to a dead textbook. Get out Eventually, you should be able to see patterns in the
on campus and apply your knowledge to the flora and fauna professors style of questions, your way of thinking through
you encounter. Practice identifying and classifying problems and your test preparation. Then you can use
organisms. Explain the biological processes that make your these insights to plan how you will study in the future.
campus green and your classmates hungry.

ANTICIPATE EXAM QUESTIONS UNDERSTAND HOW TO ANSWER


Exams dont come out of nowhere and your instructor is nt
A BIOLOGY PAPER
trying to trick you. Rather, your examiner is using the exam to Actually, what a Biology paper requires is not a composition,
make sure youve mastered important class content. So what but keywords and phrases. Markers award you marks because
content is important and whats most likely to appear on the you state a keyword or a key phrase accurately, not because you
exam? You have a lot of clues to help you figure this out write well or a long piece. Of course, you still need to make your
l
Do you synthesise all your notesfrom the lectures and answers understandable! If you dont get the key points, you
your text? You may want to make one easily accessible study will not get any marks, no matter how much you write.
sheet. This study sheet shouldnt be packed corner to corner, However, once you write down the most important point, you
but should have key ideas and pictures to jog your 2 memory. get that point regardless of the length of your answer. With that
In designing your study sheet, focus on the information you in mind, try to figure out key words or phrases in the chapter
have yet to fully master. that you study.
l
While studying, can you provide a summary of the lecture If you find figuring out keywords difficult, try discussing the
or chapter in words and pictures without looking at your problem with your teacher or classmates. Bear in mind that
notes or the text? Redraw pictures or graphs from memory. simply memorising every word that appears on your textbook
Try to explain out loud each part of the picture or graph and brings you no benefit except a painful experience in your school
then explain how they relate to the other parts. Explain how life.

Books are the quickest and most conotant of friends, they are the most
accessible and wisest of counselors and the most patient of teachers
FNAB is a technique used to investigate superficial lumps or masses. It is a safe, minor surgical procedure. Often, a
major surgical biopsy can be avoided by performing a needle aspirative biopsy. In 1981, the first fine-needle
aspiration biopsy in the United States was done at Maimonides Medical Center, eliminating the need for surgery and
hospitalisation. Today, this method is widely used in the diagnosis of cancer and inflammatory conditions.
A fine needle aspiration biopsy (FNAB) can be done in several different ways.
FINE NEEDLE ASPIRATION BIOPSY FOR PALPABLE 1. The skin above the area to
GROWTHS A palpable growth is one that can be felt.The be biopsied is swabbed with an
antiseptic solution and draped
patient usually sits up while the doctor inserts a small hollow with sterile surgical towels.
needle with a syringe to withdraw (aspirate) fluid and cells from The skin, may be numbed
with a local anaesthetic
the growth for testing. When the needle reaches the mass, the
doctor suctions out a sample with the syringe. The doctor
repeats this procedure several times. If the mass is a cyst, the 2. The needle may be Locate the lesion,
withdrawn samples will consist mainly of fluid and the cyst may inserted and withdrawn insert the needle tip Using the X-rays,
the location of
several times
collapse, relieving pain. If the mass is solid, the samples will Reason one needle may mass was identified
consist primarily of tissue cells. By analysing the sample be used as a guide, with
the other needles placed
Apply vacuum
immediately after their withdrawal, if it is found to be a cyst then along it to achieve a
more precise position
doctor diagnoses a benign. In all other cases, fluid and tissue
samples are placed on slides and then analysed by a pathologist
Maintain vacuum and
in a lab. This biopsy may signal that cancer is present. sample different areas Sometimes, several
of the lesion by inserting passes may be needed
GUIDED FNAB FOR NON-PALPABLE GROWTHS When a back and forth to obtain enough cells
for the intricate tests
growth is small or deep to palpate (feel), the doctor must locate which the
cytopathologists
it with one of several imaging techniques. The images are taken perform
from different angles to form a virtual three- dimensional
Release the vacuum
(stereotactic) image that precisely pin points the location of the completely by releasing 4. This technique
suspicious area. The computer then uses a motor to guide a the syringes piston diagnoses a carcinoma
on the left and normalee
small hollow needle to the site to remove the samples. The cells on the right side
withdrawn samples are then analysed for the presence of cancer.

ADVANTAGES OF FNAB OVER SURGICAL OPERATIONS


(a) Together with clinical-biochemical evalution, echo-guided FNAB
remains the first-line diagnostic test in the management of 3. Cells are withdrawn by aspiration
with a syringe and spread on a glass
thyroid nodules. slide. This slide is viewed by a
microscope equipped with a camera Carcinoma Normals
(b) It is safe and simple.
(c) It can be performed as an outpatient procedure and it is well tolerated by patients.
(d) In the present managed care environment, it also proves cost-effective.
(e) It is recommended as a first line of investigation in palpable head and neck masses.
1. The strand of DNA acting as template for mRNA 6. If 50 J energy is trapped by wheat plant. Then, how
transcription is much energy will be available to eagle as food in the
I. coding strand following food chain?
II. non-coding strand wheat plant mice snake eagle
III. sense strand (a) 0.05 J (b) 0.005 J (c) 6.25 J (d) 0.5 J
IV. anti sense strand 7. The morphological self incompatibility occurs in
Choose the correct one from the codes given below flowers having heterostyly. It promotes
(a) I and II (b) II and IV (c) I and III (d) II and III (a) Self-pollination (b) Cross-pollination
2. Consider the statement given below and choose the (c) Both (a) and (b) (d) None of these
correct option from the given codes. 8. Assisted reproductive technology IVF, involves
I. Menstrual cycle is present in all mammals. transfer of
II. Menstrual cycle is present in all primates. (a) ovum into the Fallopian tube
III. Oestrous cycle occurs in all mammals. (b) zygote into the Fallopian tube
IV. Most mammals are ovoviviparous. (c) zygote into the uterus
(a) I, II and IV (b) II and III (d) embryo with 16 blastomeres into the Fallopian tube
(c) I, II, III and IV (d) Only II 9. Five kingdom system of classification suggested by RH
3. A DNA molecule has radioactive thymidine. It is Whittaker is not based on
allowed to replicate in a medium containing simple (a) the presence or the absence of a well defined nucleus
thymidine (without radioactivity). In how many (b) mode of nutrition
strands radioactive thymidine will be present after (c) mode of reproduction
three replications? (d) complexity of body organisation
(a) 4 (b) 8 (c) 1 (d) 2 10. Consider the following statements with reference to a
4. A human male produces sperms with the genotype normal human being.
AB, Ab, aB and ab pertaining to the diallelic characters I. Human saliva is slightly alkaline.
in equal proportions. What is the corresponding II. An adult human may secrete 1-1.5 L of saliva
genotype of this person? per day.
(a) AABB (b) AaBB (c) AaBb (d) AABb III. Saliva is secreted by six pairs of salivary glands in
human beings.
5. The ovule in which the funicle, chalaza and micropyle IV. The salivary enzyme (ptyalin) breaks down cooked
lie in one vertical plane is called starch into maltose.
(a) amphitropous (b) anatropous Which of the statements given above are correct?
(c) campylotropous (d) orthotropous (a) I, II and III (b) III and IV (c) I, III and IV (d) I, II, III and IV

25
11. Cryptorchidism is a condition of testes in which it is 21. Geitonogamy involves
(a) unable to descend in scrotal sac (a) fertilisation of a flower by the pollen from another flower of
(b) unable to produce sperms the same plant
(c) has been surgically removed (b) fertilisation of a flower by the pollen from the same flower
(d) remain undeveloped (c) fertilisation of a flower by the pollen from a flower of another
plant in the same population
12. Archaebacteria differ from eubacteria in
(d) fertilisation of a flower by the pollen from a flower of another
(a) cell membrane structure
plant belonging to a distant population
(b) mode of nutrition
(c) cell shape 22. Dwarf pea plant was treated with gibberellic acid, it
(d) mode of reproduction became tall, as tall plants. If these pea plants are
crossed with pure tall plants, what will be the
13. Which character is not similar in all living organisms?
phenotypic ratio in F1 -generation.
(a) Similar genetic code of amino acids
(b) Energy storage in ATP (a) 50% tall and 50% dwarf plants
(c) Type of proteins in the body (b) 75% tall and 25% dwarf plants
(d) Protein synthesis in ribosome (c) All dwarf plants
(d) 100% tall plants
14. Haploids are more suitable for mutation studies than
diploids. This is because
23. Consider the following five statements.
I. In Equisetum, the female gametophyte is retained
(a) haploids are reproductively more stable than diploids
on the parent sporophyte.
(b) mutagens penetrate in haploids more effectively than in
diploids II. In Ginkgo, male gametophyte is not independent.
(c) haploids are more abundant in nature than diploids
III. The sporophyte in Riccia is more developed than
that in Polytrichum.
(d) all mutations whether dominant or recessive are expressed
in haploids IV. Sexual reproduction in Volvox is isogamous.
V. The spores of slime moulds lack cell walls.
15. Which of the following is a hormone releasing
How many of the above statements is/are correct?
Intra Uterine Device (IUD)? (a) Four (b) One (c) Two (d) Three
(a) Multiload 375 (b) LNG 20
(c) Cervical cap (d) Vault
24. Failure of a pair of homologous chromosomes to
separate in anaphase-I of meiosis is referred to as:
16. Cyathium inflorescence shows (a) Non-disjunction (b) Disjunction
(a) scorpoid cyme showing central female, many peripheral (c) Transformation (d) Transduction
male flowers
(b) scorpoid cyme showing central male, many peripheral 25. Which of the following hormones are secreted by
female flowers human placenta?
(c) dichasial cyme showing two whorls, one of male and I. hCG II. hCS III. hCT
another of female flowers (a) I and II (b) I and III (c) II and III (d) I, II and III
(d) dichasial cyme showing two whorls of 3-9 flowers 26. Consider the following four statements.
17. Which of the following product is obtained I. The sporophyte in liverworts is more elaborate than
commercially from Monascus purpureus? that in mosses.
(a) Citric acid II. Salvinia is heterosporous.
(b) Blood cholesterol lowering statins III. The life-cycle in all seed-bearing plants is diplontic.
(c) Ethanol IV. In Pinus, male and female cones are borne on
(d) Streptokinase for removing clots from the blood vessels different trees.
18. Which one of the following is gaseous biogeochemical The two wrong statements together are
cycle in ecosystem? (a) I and III (b) I and IV (c) II and IV (d) I and II
I. Sulphur cycle II. Phosphorus cycle 27. Pheretima and its close relatives derive nourishment
III. Nitrogen cycle IV. Carbon cycle from
Choose the correct one from the codes given below. (a) soil insects
(a) I, II and IV (b) II and IV (b) small pieces of fresh fallen leaves of maize, etc
(c) I, II and III (d) III and IV (c) sugarcane roots
(d) decaying fallen leaves and soil organic matter
19. Which of the following is not the modification of tap
root system? 28. Which of the following combination represents the
(a) Napiform (b) Fusiform vector of cloning capacity of DNA from smaller to bigger
(c) Tuberous (d) Bulbil (a) BAC, YAC, plasmid, cosmid
20. Role of nucleus in plant morphogenesis was studied by (b) Plasmid, BAC, YAC, cosmid
(a) Hammerling (b) Melchers (c) YAC, plasmid, BAC, cosmid
(c) Khorana (d) Robert Brown (d) Plasmid, cosmid, BAC, YAC

26
29. Germinal epithelium of ovary has Codes
(a) cuboidal cells (b) columnar cells A B C D A B C D
(c) squamous cells (d) stratified cells (a) 2 4 3 1 (b) 3 2 4 1
(c) 4 2 1 3 (d) 1 2 3 4
30. A fall in Glomerular Filtrate Rate (GFR) activates
(a) adrenal medulla to release adrenaline
37. Hing of commerce is obtained from
(a) diatoms (b) waste petroleum product
(b) posterior pituitary to release vasopressin
(c) resinous exudate from root (d) it is not a plant production
(c) adrenal cortex to release aldosterone
(d) juxtaglomerular cells to release renin 38. An organism used as a biofertilizer for raising Cajanus
cajan crop.
31. Consider the following statements and choose the
(a) Nostoc (b) Azospirillum
correct code.
(c) Rhizobium (d) Azotobacter
I. Allium sativum and Allium cepa belongs to the
family Solanaceae. 39. In gynandromorph,
II. Tomato belongs to Liliaceae family. (a) Some cells of body contain XXY and some cells with
III. Colchicum autumnale belongs to Liliaceae family. genotype XO
IV. Pneumatophores are seen in Rhizophora. (b) Some cells of body contain XX and some cells with
Codes genotype XY
(a) I, II and III (b) III and IV (c) All cells with genotype XXY
(c) I, III and IV (d) I, II, III and IV (d) All cells with genotype XX
32. Select the correct option describing gonadotropin 40. Which of the following is not under the control of
activity in a normal pregnant female. dictyosomes?
(a) High level of FSH and LH stimulates the thickening of I. Depositon of cell wall material
endometrium II. Cell plate formation
(b) High level of FSH and LH facilitate implantation of the III. Extracellular secretion
embryo IV. Autophagy
(c) High level of hCG stimulates the synthesis of oestrogen and (a) I, II and IV (b) II, III and IV (c) IV only (d) I and III
progesterone 41. Which of the following is responsible for peat
(d) High level of hCG stimulates the thickening of endometrium formation?
33. Name the type of aestivation when sepal or petals in a (a) Marchantia (b) Sphagnum (c) Funaria (d) Riccia
whorl just touch one another at the margin without 42. Apomixis is a special mechanism to produce seeds
overlapping without fertilisation. Choose the correct statement
(a) twisted aestivation (b) imbricate aestivation regarding apomixis.
(c) valvate aestivation (d) vexillary aestivation I. It is a form of asexual reproduction which does not
34. Antigen binding site in an antibody is found between involve syngamy and meiosis.
(a) two light chains II. Adventive embryony is a type of apomixis when one
(b) two heavy chains or more embryos are formed.
(c) one heavy and one light chain III. Embryo develops from the separate embryo sacs
(d) either between two light chains or between one heavy and present in the same ovule.
one light chain depending upon the nature of antigen (a) Only I (b) II and III (c) Only II (d) I and II
35. Stages in menstrual cycle are 43. Synthesis of one molecule of glucose requires
(a) recovery phase and proliferative phase (a) 6 CO 2, 18 ATP and 12 NADPH
(b) proliferative phase and secretory phase (b) 6 CO 2, 12 ATP and 18 NADPH
(c) proliferative, secretory and menstrual phase (c) 6 CO 2; 30 ATP and 12 NADPH
(d) recovery phase, secretory phase and phase of menstrual flow (d) 6 CO 2, 38 ATP and 12 NADPH

36. Some hormones are given in column I. Match the 44. Match the following and choose the correct code.
source and function with column II and choose the List I List II
correct code. A. One gene-one polypeptide 1. Crick
Column I Column II B. Genetic code was discovered 2. Temin and Baltimore
by
A. Oxytocin 1. Pineal gland, regulate the circadian
C. Reverse transcription 3. Beadle and Tatum
behaviour
D. Semiconservative DNA 4. Meselson and Stahl
B. Progesterone 2. Posterior pituitary, facilitates birth replication
C. Thymus 3. neck region, proliferation and
regulation of T-lymphocyte Codes
A B C D A B C D
D. Melatonin 4. Gonads, maintains pregnancy and (a) 2 3 1 4 (b) 3 1 4 2
uterus wall thickening (c) 1 3 2 4 (d) 3 1 2 4

27
45. Select the taxon mentioned that represents both 54. Consider the following features about climax
marine and freshwater species community.
(a) Ctenophora (b) Echinodermata I. Rapidly keeps on changing to reach equilibrium
(c) Cephalochordata (d) Cnidaria II. Final community
46. Consider the following statements. III. End of succession
I. High amount of Escherichia coli in water is an IV. Stable
indicator of sewage or faecal pollution. Choose the correct set of features from the options given
II. Bhopal gas tragedy of 1984 took place because below.
methyl isocyanate reacted with air. (a) I and II (b) II, III and IV
III. Chipko movement was initially meant for (c) I and III (d) I, II, III and IV
protecting trees, but now it is meant for 55. World Conservation Union is the another name of
preservation of environment including habitat and (a) UNEP (b) IUCN
wildlife. (c) UNESCO (d) ICFRE
Which of the statement given above are correct? 56. Commonly used vectors for human genome
(a) I and II (b) I and III (c) II and III (d) I, II and III sequencing project are
47. Antrum is cavity of (a) Expression vectors (b) T-DNA
(a) ovary (b) Graafian follicle (c) blastula (d) gastrula (c) BAC and YAC (d) T/A cloning vectors
48. An aggregate fruit is one which develops from 57. Which of the following technique is not involved in
(a) multicarpellary syncarpous gynoecium southern hybridisation for the analysis of chromosomal
(b) multicarpellary apocarpous gynoecium DNA?
(c) complete inflorescence (a) Autoradiography (b) Electrophoresis
(d) multicarpellary superior ovary (c) Blotting (d) PCR
49. Choose the correct statement from the following and 58. Function of filiform apparatus is to
select the correct code. (a) recognise the suitable pollen at stigma
I. Naked DNA is acidic. (b) stimulate division of generative cell
II. DNA is double stranded left handed helical (c) produce nectar
structure. (d) guide the entry of pollen tube
III. Guanosine is a nucleotide.
IV. Two nucleotides are linked in 3 5 to form 59. A human female with Turners syndrome
dinucleotide. (a) has 45 chromosomes with XO
Codes (b) has one additional X-chromosome
(a) I and II (b) I and IV (c) I, II, III and IV (d) I, III and IV (c) exhibits male characters
(d) is able to produce children with normal husband
50. How parasympathetic neural signal is associated with
heart? 60. Photosynthetic bacteria have
(a) Both heart rate and cardiac output increases (a) pigment system-I
(b) Heart rate decreases but cardiac output increases (b) pigment system-II
(c) Reduces both heart rate and cardiac output (c) Both (a) and (b)
(d) Heart rate is increased without affecting the cardiac output (d) Some other kind of pigments P890
51. A health disorder myxoedema that results from the 61. Taxonomic hierarchy was introduced by
deficiency of thyroxine in adults leads to (a) Ernst Mayer (b) Linnaeus
I. A low metabolic rate. (c) Aristotle (d) RH Whittaker
II. Increase in body weight. 62. Yellow corpus luteum occurs in a mammal in
III. Tendency to retain water in tissues.
(a) heart to initiate heartbeat
IV. Rise in body temperature.
(b) skin to function as pain receptor
Choose the correct code. (c) brain and connects cerebral hemisphere
(a) I and IV (b) IV only (d) ovary for secretion of progesterone
(c) II and IV (d) I, II, III and IV
63. Cry II Ab and Cry I Ab produce toxins that control
52. Which of the following is an example of sex-linked (a) cotton bollworms and corn borer respectively
disease?
(b) corn borer and cotton bollworms respectively
(a) Colour blindness (b) AIDS
(c) tobacco budworm and nematode respectively
(c) Syphilis (d) Gonorrhoea
(d) nematodes and tobacco budworms respectively
53. Which of the following is an analogous structure?
64. The first recombinant DNA was constructed by
(a) Wings of bat and wings of pigeon
(a) Paul Berg
(b) Gills of prawn and lungs of man
(b) Stanley Cohen and Herbert Boyer
(c) Thorns of Bougainvillea and tendril of Cucurbita
(c) Har Gobind Khorana
(d) Flippers of dolphin and legs of horse
(d) James D Wateson

28
65. Ischihara chart is used to detect (b) Satellite DNA occurring as highly repeated short DNA
(a) tuberculosis (b) eye sight segments
(c) colour blindness (d) diabetes (c) The relative proportions of purines and pyrimidines in DNA
(d) The relative difference in the DNA occurrence in the blood,
66. Given below is a simplified model of phosphorus skin and saliva
cycling in a terrestrial ecosystem with four blanks
(A-D). Identify the blanks. 74. Examine the figure given below and select the right
option giving all four parts (A, B, C and D) correctly
Consumers C identified.

D A
A
B
uptake
Soil solution

B Run-off
C
D
A B C D
(a) Rock minerals Detritus Litter fall Producers
(a) AArchegoniophore BFemale thallus
(b) Litter fall Producers Rock minerals Detritus
CGemma cup DRhizoids
(c) Detritus Rock minerals Producers Litter fall
(b) AArchegoniophore BFemale thallus DFoot
(d) Producers Litter fall Rock minerals Detritus
CBud
67. The element required for the activation of DNA and (c) ASeta CProtonema BSporophyte DRhizoids
RNA is (d) AAntheridiophore BMale thallus
(a) Ca 2+ (b) Cu2+ (c) Mg 2+ (d) K+ CGlobule DRoots
68. Correct sequence of hormone secretion from
beginning of menstruation is
75. If 6 n is the number of genotypes, where n is the number
of different chromosome pairs with each carrying one
(a) FSH, progesterone, oestrogen
heterozygous alleles, what would be the dif ferent
(b) oestrogen, FSH, progesterone
genotypes produced by garden pea plant?
(c) FSH, oestrogen, progesterone
(d) oestrogen, progesterone, FSH (a) 614 (b) 612
(c) 6 7 (d) 610
69. Which of the following is wrongly matched? Hint Pea plant have 7 pairs of contrasting character
(a) Transcription writing information from DNA to t RNA
(b) Translation using information in mRNA to make protein 76. In the absence of fertilization, corpus luteum
(c) Repressor protein binds to operator to stop enzyme (a) stops secreting progesterone
synthesis (b) changes to corpus albicans
(d) Operon structural genes, operator and promoter (c) starts producing progesterone
(d) starts producing hCG
70. Which enzyme/s will be produced in a cell in which
there is a non-sense mutation in the lac y gene? 77. Arrange the following option in ascending order of
(a) -galactosidase their BOD value.
(b) Transacetylase I. Sample of highly polluted pond water.
(c) Lactose permease II. Sample from unpolluted pond water.
(d) Lactose permease and transacetylase III. Distilled water.
(a) III, I, II (b) II, III, I
71. Which of the following is a characteristic feature of
(c) III, II, I (d) I, III, II
dinoflagellates?
(a) They are golden-brown coloured 78. Consider the statement given below and choose the
(b) They are motile and unicellular correct code.
(c) Have bioluminescence I. Recessive trait can only be expressed in
(d) All of the above homozygous condition.
II. Recessive trait can only be expressed in
72. Placenta and pericarp are both edible portion in heterozygous condition.
(a) apple (b) banana III. Dominant trait is expressed in homozygous
(c) tomato (d) potato
condition.
73. DNA fingerprinting is used for the detection of IV. Dominant trait cannot be expressed in
criminals, paternity test, etc. What is the basis of DNA heterozygous conditions.
fingerprinting? Codes
(a) The relative amount of DNA in the ridges and grooves of the (a) I, II and III (b) Only I
fingerprint (c) II and IV (d) II, III and IV

29
79. Devonian period of Palaeozoic era is popularly 85. Which of the following is not true about aves?
known as I. Birds are uricotelic with mesonephric kidney.
(a) age of invertebrates II. Bones are pneumatic and bone marrow present.
(b) age of fishes III. Skin is dry without glands except oil gland or green
(c) age of amphibians glands.
(d) age of terrestrial land plants IV. Fertilization is internal and are viviparous.
(a) I and II (b) I, II and IV (c) II and IV (d) I, III and IV
80. At which stage of HIV infection does one usually
shows symptoms of AIDS? 86. Match the following columns.
(a) Within 15 days of sexual contact with an infected person Column I Column II
(b) When the infected retrovirus enters host cells A. Porogamy 1. Pollen tube enters by piercing the
(c) When HIV damages large number of helper T-lymphocytes integument.
(d) When the viral DNA is produced by reverse transcriptase B. Chalazogamy 2. Pollen tube enters through
micropylar end.
81. Which of the following statement is not true about C. Mesogamy 3. Pollen tube enters through
Schwann cells? chalazal end.
I. Surround axon of myelinated nerve fibre.
II. Support muscle fibres. Codes
A B C A B C
III. Found in Haversian system of bones.
(a) 2 3 1 (b) 1 2 3
IV. Form basement membrane of epithelium. (c) 3 2 1 (d) 1 3 2
Choose the correct code from the codes given below. 87. A man whose father was colourblind marries a woman,
(a) Only I who had a colourblind mother and normal father. What
(b) I, II and IV percentage of male children of this couple will be
(c) II, III and IV colourblind?
(d) I, II, III and IV (a) 100% (b) 75% (c) 50% (d) 25%
82. In an accident a persons hypothalamus region is 88. If Mendel had studied the seven traits using a plant
injured. What would most likely disrupt with 12 chromosomes instead of 14, in what way would
(a) short term memory his interpretation have been different?
(b) executive function such as decision making (a) He could have mapped the chromosomes
(c) regulation of body temperature (b) He would have discovered blending or incomplete
(d) coordination during locomotion dominance
83. Which of the following is true about phenylketonuria (c) He would not have discovered the law of independent
(PKU)? assortment
(a) They lack phenylalanine hydroxylase to change (d) He would have discovered sex linkage
phenylalanine to tyrosine 89. Which one of the following statement is correct?
(b) Accumulation of phenylalanine and its metabolites damage (a) The seed in grasses is not endospermic
the brain and causes the disease (b) Mango is a parthenocarpic fruit
(c) The heterozygous individual is normal but carrier (c) A proteinaceous aleurone layer is present in maize grain
(d) All of these (d) A sterile pistil is called a staminode
84. A few normal seedlings of tomato were kept in a dark 90. Sacred groove is one of the great aspect of in situ
room. After a few days, they were found to have conservation. It is useful in
become white-coloured like albinos. Which of the (a) year round flow of water in rivers
following terms will you use to describe them? (b) conserving rare and threatened species
(a) Mutated (b) Embolised (c) generating environmental awareness
(c) Etiolated (d) Defoliated (d) preventing soil erosion

Answers
1. (b) 2. (d) 3. (d) 4. (c) 5. (d) 6. (a) 7. (b) 8. (b) 9. (a) 10. (a)
11. (a) 12. (a) 13. (c) 14. (d) 15. (b) 16. (a) 17. (b) 18. (d) 19. (d) 20. (a)
21. (a) 22. (d) 23. (b) 24. (a) 25. (d) 26. (b) 27. (d) 28. (d) 29. (a) 30. (d)
31. (b) 32. (c) 33. (c) 34. (c) 35. (c) 36. (a) 37. (c) 38. (c) 39. (b) 40. (c)
41. (b) 42. (d) 43. (a) 44. (d) 45. (d) 46. (b) 47. (b) 48. (b) 49. (b) 50. (c)
51. (d) 52. (a) 53. (b) 54. (b) 55. (b) 56. (c) 57. (d) 58. (d) 59. (a) 60. (d)
61. (b) 62. (d) 63. (a) 64. (b) 65. (c) 66. (c) 67. (c) 68. (c) 69. (a) 70. (a)
71. (d) 72. (c) 73. (b) 74. (a) 75. (c) 76. (b) 77. (c) 78. (b) 79. (b) 80. (c)
81. (c) 82. (c) 83. (d) 84. (c) 85. (c) 86. (a) 87. (d) 88. (c) 89. (c) 90. (b)

30
AIPMT & AIIMS

T
PM AIIMS
Questions to Measure Your Problem Solving Skills
This test consists of 60 questions and each question is allotted 4 marks for correct response.
Instructions

Candidates will be awarded marks as stated above for correct response of each question. 1/4 marks will be deducted for indicating incorrect
response of each question. No deduction from the total score will be made if no response is indicated for an item in the answer sheet.
There is only one correct response for each question. Filling up more than one response in any question will be treated as wrong response
and marks for wrong response will be deducted according as per instructions.

1. One of the parents in a cross has mutation in its 6. Choose the correct statement about Bt toxin.
mitochondrial DNA. In that cross, that parent is taken (a) The concerned Bacillus had antitoxin
as a male. During segregation of F2 progenies, that (b) Bt protein exists as active toxins in the Bacillus
mutation is to be found in (c) It is used to kill herbivores like deer and rabbit
(a) One-third of the progenies (d) The activated toxin enters the ovaries of the pest to sterilise
(b) Fifty per cent of the progenies it and thus, prevents its multiplication
(c) None of the progenies 7. Which of the following example represents the
(d) All the progenies autonomic nervous system?
2. A short length of DNA molecule has 60 thymine and (a) Swallowing of food (b) Pupillary reflex
60 guanine bases. The total number of nucleotide in (c) Knee-jerk response (d) Peristalsis of the intestine
the DNA fragment is 8. In the resting state of the neural membrane, diffusion
(a) 100 (b) 240 (c) 120 (d) 60 due to the concentration gradients, if allowed, would
3. Which statement regarding SARS (Severe Acute drive.
Respiratory Syndrome) is true? (a) K + into the cell (b) K + and Na + out of the cell
(a) It is caused by a variant of Pneumococcus pneumoniae (c) Na + into the cell (d) Na + out of the cell
(b) It is caused by a variant of the common cold virus
(corona virus)
9. Countercurrent mechanism helps to maintain a
(c) It is an acute form of asthma
concentration gradient. This gradient helps in
(d) If affects non-vegetarian much faster than the vegetarians (a) easy passage of water from medullary interstitial fluid to
collecting tubule and thereby diluting urine.
4. If a plant produces flowers only when exposed to (b) easy passage of water from medulla to collecting tubule
alternating periods of 5 hrs light and 3 hrs dark in a and thereby concentrating urine.
24 hrs cycle, then the plant should be (c) inhibition of passage of water between the collecting tubule
(a) short day plant (b) day neutral plant and medulla and so isotonic urine is formed.
(c) short-long day plant (d) long day plant (d) easy passage of water from collecting tubule to interstitial
5. For growing a plant in a xerophytic condition, what fluid and thereby concentrating urine.
type of adaptation plant requires. 10. What is the function of germ pore?
(a) The presence of spine (a) Release of male gametes
(b) The presence of stipular leave (b) Emergence of radical
(c) The absence of stomata (c) Initiation of pollen tube
(d) The presence of long tap root system (d) Absorption of water and seed germination

31
AIIMS SCALE UP

11. Choose the correct statement. 20. Gynandrous condition shows


(a) Leaf abscission is caused by gibberellin (a) all anthers are united except filament
(b) The rosette habit of cabbage can be changed by (b) adhesion of petals with stamens
application of gibberellic acid (c) adhesion of stamens with carpel
(c) NAA is a synthetic auxin (d) stamens are united throughout their whole length.
(d) Both (b) and (c) 21. In case of haemophilia, if the carrier daughter marries
12. The reason behind lowering of rate of transpiration a normal man, then among their daughters
during storm is (a) 25% are carrier only
(a) low temperature during storms (b) 50% are carrier and 50% are haemophilic
(b) high velocity of winds (c) 50% are carrier only
(c) the presence of moisture in winds (d) 75% are carrier and 25% are haemophilic
(d) All of the above 22. Consider the case of a family with five children. It
13. If cell A with OP = 5 and TP = 4 is surrounded by cells shows the inheritance of attached ear-lobes as
with OP and TP = 1, what will be the action of water opposed to the free ones. The squares represent the
movement? male individuals and circles represent the female
(a) Water will move up (b) Water will not move up individuals.
(c) From other cells to cell A (d) From, cell A to other cells
14. An example of competitive inhibition of an enzyme is
the inhibition of
Free
(a) succinic dehydrogenase by malonic acid Attached ear-lobe
(b) cytochrome oxidase by cyanide ear-lobe

(c) hexokinase by glucose-6-phosphate


(d) carbonic anhydrase Which one of the following conclusions drawn is correct?
15. Which of the following is not a lateral meristem? (a) The parents are homozygous recessive
(a) Intercalary meristem (b) The trait is Y-linked
(b) Phellogen (c) The parents are homozygous dominant
(c) Intrafascicular cambium (d) The parents are heterozygous
(d) Interfascicular cambium 23. Choose the incorrect statement which is given below.
16. Match column I with column II and select the correct (a) Cushing syndrome is caused due to the increased
option from the codes given below. production of cortisol
(b) Small in size and variable in structure is the feature of fatty
Column I (Organelle) Column II (System) acids based hormones
A. Mitochondria 1. Light harvest system (c) Ecdysone helps in metamorphosis in insects
B. Chloroplast 2. Cytoskeleton (d) Calcium level decreases in the blood due to hyposecretion
C. Endoplasmic reticulum 3. Electron transport system of parathormone
D. Peroxisome 4. EMP system 24. The further treatment of fibres with saturated salt
5. Photorespiratory system solutions removes actin and its two associated proteins
Codes tropomyosin and troponin and also destroy the
A B C D A B C D I-bands. This shows that the thin filaments are made
(a) 3 2 1 4 (b) 3 1 2 5 up of protein/s
(c) 4 1 3 2 (d) 3 2 4 1 (a) tropomyosin and troponin
17. In an accident a persons abducens nerve is injured. (b) myosin
Which one of the following functions will get affected? (c) actin
(a) Movement of neck (d) tropomyosin + troponin + actin
(b) Movement of the eye ball 25. The concept of chemical evolution is based on the
(c) Swallowing (a) effect of solar radiation on chemicals
(d) Movement of hand and leg (b) crystallisation of chemicals
18. The first electron acceptor from an excited chlorophyll (c) interaction of water, air and clay under intense heat
molecule of photosystem-II is (d) possible origin of life by combination of chemicals under
(a) ferredoxin (b) iron-sulphur protein suitable environmental conditions
(c) plastoquinone (d) cytochrome 26. Which of the following is wrongly matched?
19. Chicory powder, which is mixed with the coffee (a) RNA interference Gene silencing
powder is obtained from (b) Calcitonin To treat infertility
(a) seeds (b) roots (c) Removal of C-peptide Maturation of proinsulin
(c) leaf (d) stems (d) Hybridoma technology Monoclonal antibodies

32
AIIMS SCALE UP

27. Match the types of vascular bundles listed under Diseases Glands
Column I with examples given under Column II and A. Addisons disease 1. Pituitary
select the correct option from the codes given below. B. Tetany 2. Thyroid
Column I Column II C. Acromegaly 3. Adenal cortex
(Vascular bundle) (Example) d. Myxedema 4. Parathyroid
A. Collateral and open 1. Cucurbita stem Codes
B. Radial 2. Ferns A B C D A B C D
C. Bicollateral 3. Maize root (a) 3 4 1 2 (b) 1 3 2 4
D. Concentric 4. Sunflower (c) 3 4 2 1 (d) 3 2 1 4
5. Maize stem 34. Which of the following elements is an activator for both
Codes ribulose bisphosphate carboxylate oxygenase and
A B C D A B C D Phosphoenol pyruvate carboxylase in photosynthetic
(a) 4 3 1 2 (b) 5 3 2 1 carbon fixation?
(c) 4 2 3 1 (d) 3 1 2 4 (a) Zn2 + (b) Mn2 + (c) Ca 2 + (d) Mg 2 +
28. Select one of the following pairs of important features 35. A set of parasites and their respective vector/
distinguishing Gnetum from Cycas and Pinus and intermediate host are given. Match the following and
showing affinities with angiosperms select the correct answers with the help of the codes
(a) perianth and two integuments given below.
(b) the presence of vessel elements and the absence of
archegonia Column I Column II
(c) embryo development and apical meristem A. Dracumuculus medinensis 1. Sandfly
(d) the absence of resin duct and leaf venation B. Schiotosoma haematobium 2. Cyclops
C. Leishmania donovani 3. Cattle
29. The smooth sustained contraction of a muscle due to D. Taenia saginata 4. Snails
the fusion of many twitches is called
(a) tendon (b) tetanus Codes
(c) rigor mortis (d) vigor mortis A B C D A B C D
(a) 2 4 1 3 (b) 2 3 1 2
30. In C4 plants showing Kranz anatomy, which of the (c) 4 2 1 3 (d) 1 3 2 4
following is correct?
(a) Bundle sheath cells agranal and mesophyll cells granal 36. Dimorphism of nuclei is unique to which protistan
(b) Bundle sheath cells granal and mesophyll cells agranal group
(c) Both are agranal (a) ciliata (b) sarcodina (c) sporozoa (d) flagellata
(d) Both are granal 37. Which of the following is a characteristic feature of
31. During labelling the diagram of human left limb, a mammals without any exception?
student has done some mistakes in labelling. The (a) Mammals are viviparous and have biconcave RBCs
wrongly labelled bones in the figure are (b) Heterodont teeth and 12 pairs of cranial nerves
(c) Muscular diaphragm and milk producing glands
Femur (d) Extra-abdominal testes and a four chambered heart
38. The female sex organ (archegonium) in the case of
Tibia Anthoceros (bryophytes)
Fibula
(a) is always on stalks
Tarsals
(b) is always on the surface
Phalanges (c) is always placed ventrally
(d) is almost completely embedded in the thallus tissue and
the top of neck is exposed
(a) tibia and tarsals
(b) femur and fibula 39. Hartig net is formed due to
(c) fibula, phalanges and tibia (a) aggregation of mycelium
(d) tarsals and femur (b) association of fungal hyphae with host root
(c) intermingling of cytoplasm and nuclei of + mating type
32. In Blackmanns Law of Limiting Factors the rate of
hyphae and ve mating type hyphae
photosynthesis continues to increase with the
(d) All of the above
successive increase in the amounts of
(a) carbon dioxide, light and temperature 40. Mark the hypertonic solution.
(b) temperature, light and carbon dioxide (a) When osmotic pressure of solution is zero.
(c) light, temperature and carbon dioxide (b) Osmotic pressure of cell sap and solution is equal.
(d) light, carbon dioxide and temperature (c) Osmotic pressure of solution is lower than that of the cell sap.
(d) When osmotic pressure of solution is higher than that of the
33. Pick up the correct match in relation to glands and cell sap.
diseases associated with them.

33
Directions (Q. Nos. 41-60) Read the statements for Reason The primary consumers take in solar energy in
Assertion and Reason carefully and mark the correct option the form of food from plants.
out of the options given below. 51. Assertion Leghaemoglobin is a compound of iron and
(a) Both Assertion and Reason are correct and Reason is the is present in the root nodules of leguminous plants.
correct explanation of Assertion.
Reason It acts as oxygen scavenger and forms a
(b) Both Assertion and Reason are correct, but Reason is not temporary compound by combining with oxygen.
the correct explanation of Assertion.
(c) Assertion is correct, but Reason is incorrect. 52. Assertion Hypnospore are the thick walled resting
(d) Both Assertion and Reason are incorrect spores produced in some algae.
41. Assertion Homogeniser and ultracentrifuge are the Reason The megaspore of Pinus are also called
devices for separating various cell organelles. hypnospore.
Reason The radioactive substances used in biological 53. Assertion Appearance of pubic hair in human beings
studies are called tracer elements. indicates the onset of puberty.
42. Assertion Enzymes differ from man-made catalyst as Reason Pubis is a bone of the pelvic girdle of
the enzymes affect fewer substrate. human beings.
Reason Enterokinase is activator for pepsin. 54. Assertion The initiation step of protein synthesis
43. Assertion Archaebacteria are the only organisms with differs in several ways in prokaryotes and eukaryotes.
exceptional plasma membrane where lipids help them Reason They both form mRNA-tRNA complex with
to survive in extreme hot, cold or methane and salt rich smaller subunit of ribosome.
conditions.
Reasons Archaebacteria are called ancient bacteria. 55. Assertion Those two pairs of genes which if present in
dominant stage, do not produce any expression but
44. Assertion The first set of reactions of photosynthesis
when present collectively in dominant stage, produce
takes place in the thylakoid, where chlorophyll and
phenotypic expression are called complementary
other pigments are located and are called light
genes.
dependent reactions.
Reason They are dependent on light Reason Inheritance of purple colour of flower in a pea
gives a phenotypic ratio of 9 : 7, as a result of selfing of
45. Assertion Chelating agents used in improving F1 -offspring.
availability of some minerals in soil are actually
electron acceptors. 56. Assertion Of all the taxa, only one that exists in nature
Reason They increase solubility of some minerals in as biologically cohesive unit is species.
acidic soil. Reason Only member of the same species can
46. Assertion Inhabitants close to very busy airports are interbreed.
likely to experience health hazards. 57. Assertion Photorespiration is the respiration which
Reason Sound level of jet aeroplanes usually exceeds occurs during the day time.
160 db. Reason Photorespiration is biochemically different
47. Assertion Serendipity refers to discoveries as a result from normal respiration involving glycolate
of thorough research. mechanism.
Reason Discovery of penicillin was not due to the 58. Assertion Progenote is considered to be the most
serendipity. ancient form of life which gave rise to archaebacteria
48. Assertion Sieve tubes in plants are formed of and eubacteria.
enucleated cells called sieve members. Reason Monerans are always unicellular.
Reason All the activities of sieve cell are controlled by
59. Assertion Positive diffusion occurs against the
nucleated parenchyma cells.
concentration gradient.
49. Assertion Process of conversion of ADP into ATP is Reason Osmosis does not require a semipermeable
called as phosphorylation. membrane.
Reason Anaerobic respiration releases more energy
than aerobic respiration. 60. Assertion The synthesis of DNA takes place in
G2 cycle of mitosis.
50. Assertion Net primary production represents food
Reason DNA is positively charged particle present in
potentially available to primary consumers and
chromosome of nucleus.
decomposers, feeding upon plants.

Answers
1. (c) 2. (b) 3. (b) 4. (d) 5. (d) 6. (a) 7. (d) 8. (c) 9. (d) 10. (c)
11. (d) 12. (b) 13. (c) 14. (b) 15. (a) 16. (b) 17. (b) 18. (c) 19. (b) 20. (c)
21. (c) 22. (d) 23. (b) 24. (d) 25. (d) 26. (b) 27. (a) 28. (b) 29. (b) 30. (a)
31. (c) 32. (a) 33. (a) 34. (d) 35. (a) 36. (a) 37. (c) 38. (d) 39. (b) 40. (d)
41. (b) 42. (c) 43. (b) 44. (a) 45. (d) 46. (a) 47. (d) 48. (c) 49. (c) 50. (c)
51. (b) 52. (c) 53. (c) 54. (c) 55. (a) 56. (a) 57. (b) 58. (c) 59. (d) 60. (d)

34
Sanubia

BASED QUESTIONS

1. Consider the diagram given below and carefully 3. Observe the figure of motor unit given below.
read the statements with reference to it. Skeletal
A muscle fibres

B Axon of
motor
C nerve
Myelin
sheath
Statements
I. C is a thin filament and made up of two F-actins.
II. The complex protein, B is distributed at regular Neuromuscular
junction
intervals of troponin.
III. A is a thick filament which is not a polymerised
protein.
Nuclei of muscle fibres
IV. The globular head of meromyosin consists of
Light Meromyosin (LMM).
Identify the correct set of statements describing it,
Choose the option with correct set of statements. from codes given below.
(a) Only III (b) Only I
I. The motor unit consists of two motor, neuron
(c) I, II and III (d) II and IV
and muscle fibres.
2. Carefully observe the diagram given below of II. Sarcolemma lying beneath the nerve endings
pectoral girdle. is called motor end plate.
A B III. Axon terminals and motor end plate together
Spine
constitute neuromotor junction.
C
IV. Motor unit transmit impulses to brain.
Codes
(a) I and IV (b) I, II and III
(c) II and III (d) I, II, III and IV
4. Carefully observe the diagram and identify the
states of sarcomere in A, B and C.
I-band
H-zone A-band
Consider the statements with reference to the
structure. A

I. A is attached to scapula for articulation with


Z-line Z-line
clavicle bone. Z-line

II. C facilitates articulation of femur.


B
III. A is formed from scapula bone.
IV. B provides surface to which tendons of muscles
attach.
Choose the correct code with incorrect set of C
statements.
(a) I, II and III (b) I, II and IV Two sarcomers
(c) III and IV (d) I, II, III and IV

36
Choose the correct option identifying the state of 7. Identify A, B, C and D in the given diagram and
sarcomeres. choose the correct option.
(a) AContracting, BRelaxed, CMaximally contracted A
(b) ARelaxed, BContracting, CMaximally contracted B C
(c) AMaximally contracted, BContracting, CRelaxed
(d) ARelaxed, BMaximally contracted, CContracting
D
5. Consider the diagram of a human limb and identify
its components A, B, C and D.
A (a) AActin binding site, BATP binding site, CHead, D-cross
arm
B (b) AActin binding site, BATP binding site, CHead, Side arm
C
(c) AActin binding site, BATP binding site, CHead DLong
arm
(d) AActin binding site, BATP binding site, CHead D-Short
arm
D
8. Given diagram shows the right pectoral girdle and
upper arm (frontal view) of human female. Identify
the labels A to G.

Choose the correct option.


(a) Allium, BIschium, CPubis, DPatella
(b) Allium, BPubis, CIschium, DPatella
(c) Allium, BPatella, CIschium, DPubis B
(d) Allium, BPatella, CPubis, DIschium
C
6. Carefully observe the diagram given below. Identify
the parts labelled.
Ball and socket joint
Clavicle D
E
Tendon

F
G

B Tendon

Select the correct option.


(a) A1st Vertebra, BScapula, CHumerus, DRadius EUlna,
A
FCarpals, GMetacarpals
C (b) AScapula, BClavicle, CHumerus, DRadius, EUlna,
FCarpals, GMetacarpals
D (c) Allium, BScapula, CHumerus, DRadius, EUlna,
FCarpals, GMetacarpals
(d) AClavicle, BScapula, CHumerus, DRadius, EUlna,
Consider the statements describing the parts and FCarpals, GMetacarpals
select the correct ones. 9. Consider the diagram given below and identify the
I. The diagram shows antagonistic arrangement structures marked A-E.
of muscles.
II. B is identified as flexor.
III. D shows pivot joint.
IV. C helps in bending of arm, therefore is also an E A
flexor.
Choose the correct option from codes below. D B
(a) I and II (b) II and IV
C
(c) III only (d) I and IV

37
Choose the correct set of option. III. Calcium ions are released into the sarcoplasm
(a) ACross bridge, BCross bridge formation, CBreakage of which bind to troponin, causing a change in its
cross bridge, DSliding, EATP shape and position.
(b) ACross bridge, BCross bridge formation, IV. Myosin cross bridges are able to bind to
CSliding/rotation, DBreaking of cross bridge, EATP exposed active sites of act in.
(c) ACross bridge, BBreaking of cross bridge,
CSliding/rotation, DCross bridge formation, EAMP Choose the correct option
(d) ACross bridge, BCross bridge formation, (a) I, II and III
CSliding/rotation, DADP, EBreaking of cross bridge (b) II and IV
(c) I and IV
10. Given below is a diagrammatic representation of
(d) I, II, III and IV
human skull. Identify A, B, C and D marked in it.
B 12. Identify the labels A, B, C and D in given diagram.
C
A
A

Vertebral
C foramen

Choose the correct option identifying the labels. A B C D


(a) AHyoid bone, BMaxilla, CFrontal bone, DParietal bone (a) Odontoid Neural spine Vertebral Centrum
(b) AHyoid bone, BMaxilla, CParietal bone, DFrontal bone process artery
(c) AMaxilla, BHyoid bone, CParietal bone, DFrontal bone (b) Articular Transverse Neural spine Centrum
(d) AParietal bone, BFrontal bone, CMaxilla, DHyoid bone process process
11. Consider the given figure showing events in muscle (c) Transverse Articular Centrum Neural
contraction. process process spine
(d) Vertebral artery Neural spine Centrum Odontoid
ATP
process

13. Consider the given diagram and identify the labels


ATP hydrolysed
ATP A to ADP and Pi A, B, C, D and E.

A
AD
P+
Pi

B
C
D B
ADP
Pi
D
ADP+
Pi

C Choose the correct option identifying the labels.


(a) Allium, BAcetabulum, CPubic symphysis, DIschium,
Which of the statements given below are true with EPubis
reference to above diagram. (b) AStapes, BAcetabulum, CPubic symphysis, DIschium,
I. Neurotransmitter acetylcholine is released as EPubis
nerve impulse reaches the end of axon and bind (c) AIncus, BAcetabulum, CPubic symphysis, DIschium,
to receptor sites of motor end plate. EPubis
II. Action potential passes from motor end plate (d) ACoccyx, BAcetabulum, CPubic symphysis,
over sarcolemma and then into the T-tubules DIschium, EPubis
and sarcoplasmic reticulum.

38
14. Identify the types of joints in given figures A, B, C Choose the correct option
and D. (a) ASternum, BVertebral column, CRibs
(b) ARibs, BVertebral column, CSternum
Humerus
(c) ARibs, BSternum, CCoccyx
(d) ASternum, BRibs, CVertebral column
Tarsals
17. Identify A, B and C in the given diagram and choose
Ulna
the correct option.
Frontal Sutures
A
Radius
Parietal
Ilium of pelvic
girdle
A
B Nasal Temporal
Carpals C
Head of Ethmoid
femur
Lacrimal
D
B
Maxilla
C

Choose the correct option Mastoid process


Mandible of temporal bone
A B C D External
(a) Pivot Saddle Gliding Hinge auditory meatus
(b) Gliding Ball and socket Saddle Condyloid A B C
(c) Hinge Gliding Pivot Saddle (a) Lacrimal Ethmoid Maxilla
(d) Condyloid Pivot Saddle Ball and socket (b) Zygomatic Sphenoid Occipital
(c) Sphenoid Zygomatic Occipital
15. Identify the labels A-G in the given diagram with (d) Ethmoid Lacrimal Zygomatic
the reference of list of hints enlisted below (I-VII).
A
18. Consider the given diagram and identify the parts
E D
A, B and C.
A

B C

G
F
Hints
I. Aband II. Iband
III. Sarcomere IV. Hzone
V. Myosin
VI. Actin, troponin and tropomyosin B

VII. Zline C
Select the correct option. Choose the correct option.
(a) IE, IID, IIIF, IVG, VB, VIC, VIIA (a) ACervical vertebrae, BCoccyx, CSacrum
(b) IE, IID, IIIC, IVG, VB, VIA, VIIF
(b) ACervical vertebrae, BCoccyx, CAtlas
(c) IE, IID, IIIF, IVG, VC, VIC, VIIB
(d) IE, IID, IIIF, IVA, VB, VIC, VIIG (c) ACervical vertebrae, BCoccyx, CAxis
(d) ACervical vertebrae, BSacrum, CCoccyx
16. From the diagram given below, identify the labels
marked A, B and C. 19. Identify A, B, C and D in the given diagram and
choose the correct option.

A
A B

B
C
C
D

39
(a) AFascicle, BMuscle fibre, CSarcolemma, DBlood 22. Consider the given diagram and identify the labels
capillary A, B and C.
(b) AMuscle fibre, BFascicle, CSarcolemma, DBlood
capillary
(c) AMuscle fibre, BFascicle, CSarcoplasm, DBlood A
capillary
(d) AMuscle fibre, BEndoplasmic reticulum, CSarcoplasm,
DBlood capillary
20. With reference to the structure of muscle, consider B

the diagram of thick filament Sternum


Myosin molecules Myosin head
C
T12
L1

Given below are few statements regarding the RIB 12


structure of thick myofilaments. RIB 11
I. Each myosin is a polymerised protein.
A B C
II. Many meromyosin constitutes one thick (a) Clavicle Costal cartilages Xiphoid process
filament (myosin) (b) Costal cartilages Body of sternum Manubrium
III. Each meromyosins tail is called heavy (c) Xiphoid process Clavicle Costal cartilages
meromyosin (HMM) and head is called light (d) Manubrium Body of sternum Xiphoid process
meromyosin (LMM). 23. Consider the given diagrams in column I and
IV. The globular head is an active ATPase enzyme choose the correct type for each of them from
and has binding sites for ATP and active sites column II.
for actin.
Choose the option with correct set of statements. Column I Column II
(a) I, III and IV (b) II and III A. (i) Pivot
(c) I, II and IV (d) I, II, III and IV
21. Carefully observe the given diagram with reference
to series of events during muscular contraction.
Half I-band One A-band Myosin filament
Z M Z B. (ii) Saddle

Actin H Z-band
filament

Z-band C. (iii) Gliding

Consider the given statements describing the steps


during the above mentioned process.
D. (iv) Hinge
I. Myosin head binds to the exposed active site on
actin to form a cross bridge.
II. The Z-line attached to these actin are also
pulled inwards thereby causing shortening of
sarcomere, also called contraction.
III. This pulls the attached actin filaments towards
the centre of A-band. Choose the correct codes.
Arrange the given steps in sequence from first A B C D
(a) (i) (iii) (ii) (iv)
to last and choose the correct option depicting
(b) (iii) (iv) (i) (ii)
it. (c) (ii) (iii) (i) (iv)
(a) I II III (b) III II I (d) (iii) (i) (iv) (ii)
(c) I III II (d) III I II

40
24. Consider the given diagram of a muscle and read II. It possess two transverse processes, a neural
the statements describing it. arch with neural canal for spinal cord
Thick Sarcomere
III. It possess pre-zygapophysis and
filament Z-band or
Krause
post-zygapophysis on anterior and posterior
membrane side respectively.
Thin IV. The central part of intervertebral disc is called
filament nucleus pulposus.
Choose the correct set of statements from option
M-Band
I-Band H-Band I-Band given below.
A-Band (a) I and IV (b) II and III (c) Only I (d) I, II, III and IV
Statements 27. Consider the diagram showing muscle structure
I. In the centre of each I-band is an elastic fibre below.
(Z-line) which bisects it. A-band I-band H-zone I-band A-band
II. Thin filament are firmly attached to the Z-line.
Myofibril
III. M-line is a fibrous membrane in the middle of
Z-line Z-line
A-band.
Sarcomere
IV. A sarcomere comprises one full A-band and two
A-band
half I-bands. I-band I-band
M-zone
Choose the incorrect statements from the given
options.
(a) I and III (b) II and IV
(c) Only II (d) None of these
25. Consider the diagram of thin myofilament. Cross
H-zone
Thick Z-line
Z-line myofilament
Tropomyosin bridges
Thin
myofilament

Identify the events that occur during skeletal


muscle contraction:
Actin
I. I band shortens
Now, read carefully the statements about molecular II. A-band shortens
arrangement of actin and myosin in the filament.
III. H-zone shortens
I. Each actin (thin filament) is made up of 2F
IV. Sarcomere contract
(filamentous) actins.
V. ATP changes to ADP and Pi.
II. F-actin is the polymer of G (globular) actin.
III. 2F actins are twisted into a helix. Choose the option with incorrect events.
IV. Two strands of tropomyosin protein lies in the (a) Only I (b) Only III (c) IV and V (d) Only II
grooves of F-actin. 28. Consider the diagram of human pelvic girdle and
Choose the correct set of statements from the codes read the statements describing it.
Sacral
given bolow. canal
(a) I and II (b) III and IV
A
(c) I and IV (d) All except IV
Coccyx
26. Consider the statements given below with reference B D
to the diagram of typical vertebrum.
Transverse Neural
process spine Femur

Neural
canal
Articulating
surface
Centrum

Statements
I. It has a acoelous centrum

41
Statements 30. Consider the diagrams showing various stages of
I. It is formed by two innominate bones. muscles during its contraction
II. Each innominate bone comprises of three I-band A-band I-band
M-line
separate bones, i.e. ilium, ischium and pubis
III. A is a deep depression which articulates head of
Relaxed
femur. muscle
IV. B is made up of white fibrous cartilage
Z-line H-zone Z-line
Choose the correct set of statements Thick
Thin
myofilament
(a) II, III and IV (b) Only III Cross bridge myofilament
(c) I and IV (d) I, II, III and IV
29. Consider the figures described below A and B. Partially
contracted
muscle
Sutures

Maximally
contracted
muscle

Read the statements with reference to above


A B
mentioned stages.
I. In resting muscle fibre, the outside of
Identify the statements describing the figures A sarcolemma is negatively charged.
and B. II. Resting potential of membrane is maintained
I. A shows immovable joints. by calcium ions.
II. A have white fibrous tissue bteween the ends of III. When the tension remains the same and the
the bones. length of the muscle changes it is called
III. B shows the presence of synovial fluid that isotonic contraction.
helps in lubrication of joints IV. When the length of muscle fibres remain same
IV. B type of joints are found in pubic symphysis of and tension is increased it is called isometric
pubis contraction
Choose the code with correct set of statements. Choose the correct set of statements
(a) II and III (b) III and IV (a) I, II and IV (b) III and IV
(c) I, II and IV (d) I, II, III and IV (c) Only I (d) Only II

Answers
1. (b) 2. ( b) 3. (c) 4. (b) 5. (b) 6. (a) 7. (a) 8. (d) 9. (b) 10. (d)
11. (d) 12. (b) 13. (a) 14. (b) 15. (a) 16. (d) 17. (c) 18. (d) 19. (a) 20. (c)
21. (c) 22. (a) 23. (b) 24. (d) 25. (d) 26. (d) 27. (d) 28. (d) 29. (c) 30. (b)

AIIMS MBBS 2016 on 29th May,


Registrations begin from January last
AIIMS MBBS 2016 popularly known as All India Institute of Medical Sciences is a national level
undergraduate medical entrance exam conducted to offer admission in MBBS programme. AIIMS
entrance exam is conducted by All India Institute of Medical Sciences (New Delhi). Those applicants who
have an interest shall fill the application form from the last week of January, 2016. On the basis AIIMS
MBBS 2016 exam the candidates can apply for admission to 672 MBBS seats in 7 AIIMS institutions
located across India. Among 672 MBBS seats in AIIMS, 72 seats are in AIIMS-Delhi and the remaining 600
seats are in 6 new AIIMS institutions. This exam shall be conducted in 154 cities across the nation. AIIMS
2016 MBBS entrance exam shall be held on the 29th of May, 2016.

42
PRODUCTIVE FACT BITES

BIOTECHNOLOGY
Applications of Biotechnology
l
The plants obtained through genetic engineering containing a gene, Applications in Plant Biotechnology
usually from an unrelated organism are known as transgenic plants. l
The silencer sequence associated with the gene for alcohol
l
Transgenic plants have proved to be extremely valuable tools in dehydrogenase 1 (Adh 1) of maize is an example of negative
studies on plant molecular biology, regulation of gene action and regulating element. This gene is expressed only under anaerobic
identification of regulatory/promotory sequences, e.g. T-DNA and conditions, and hence only in roots. The Adh 1 enhancer sequence
transposable elements produce mutations by becoming inserted seems to suppress Adh 1 expression under aerobic conditions. Adh 1
within genes and thereby act as molecular tags for gene is expressed because the enhancer is unable to suppress its
identification and isolation. expression in the absence of O2 stimulus.
l
Transgenic plants also permit the analysis of metabolic pathways, l
Several varieties of insect resistant transgenic crops have been
studies of cis and trans-acting factors in gene function and the successful. Such resistant plants contain either a gene from the
elucidation of plant responses to environment stresses, etc. bacterium B. thuringiensis or cowpea trypsin inhibitor gene.
l
A typical plant gene has regions beginning with 5-end-promoter, l
These genes produce crystal proteins that form crystalline
enhancer/silencer, transcriptional start or cap site, an untranslated inclusions and thus are responsible for insecticidal activities. The cry
region or leader sequence, initiation codon, exons, introns, a second genes have been grouped into 16 distinct groups which either code
untranslated region and lastly a poly-A tail. for a 130 k Da or 70 kDa protein.
l
The 5 leader sequence of mRNA is the untranslated sequence lying l
The toxic function of these proteins is localised in the N-terminal
between cap site and ATG. The end of gene coding sequence is half of 130 kDa proteins. The C-terminal half of these proteins is
specified by chain termination codon TAG or one of the other two highly conserved and most likely involved in crystal formation.
nonsense codons. l
None of the truncated proteins crystallises in the typical
l
A variety of promoter sequences have been used to drive genes in bipyramidal shape of most of the 130 kDa proteins.
plant cells, such as nos (nopaline synthase), ocs (octopine synthase) l
The Cry I proteins are insecticidal to Lepidopteran insects, all the
and mas (mannopine synthase) promoters from Agrobacterium.
proteins even the Cry IA subfamily, have a distinctive insecticidal
l
35 S-RNA gene promoter of Cauliflower Mosaic Virus (CMV) is the spectrum.
most commonly used constitutive promotor both in dicots and l
The Cry II A proteins are active against both Lepidoptera and Diptera,
monocots. It is 10-40 folds more efficient than the nos promoter.
while Cry II B is specific to Diptera. Their homology with other Cry
l
The maize alcohol dehydrogenase 1 (Adh 1) promoter shows proteins is rather limited.
anaerobic induction, i.e. expression in roots. The Adh 1 promoter l
The Cry III proteins are active against Coleoptera species, while
activity in monocots is either comparable to or higher than that of
Cry IV proteins are specific to Diptera.
35 S promoter.
l
The Cyt A protein does not show any insecticidal activity but is
l
Expression of many genes is confined to specific tissues or induced
cytolytic for a variety of vertebrate and invertebrate cells. It does
by specific stimuli, such genes are called tissue specific or
not exhibit any homology with other Cry proteins.
stimulus-responsive genes respectively.
l
Truncated cry genes are used for the production of transgenic
l
Such specificities in gene expression are due to certain DNA
plants since, the level of expression of complete genes in transgenic
sequence called enhancers or silencers, either lying within or at a
plants is extremely low.
considerable distance (upto several kbs) away from the promoters
they affect.
l
The Cry I protein A genes have been successfully transferred into
tobacco, potato and tomato.
l
An enhancer may be defined as a DNA sequence which increases the
activity of a promoter gene while DNA sequences suppressing
l
The transgenic tobacco plants expressing Cry protein at about
promoter activity are called silencers. Thus, enhancer and silencer 0.004% of their total leaf protein killed all M. sexta larvae within
sequences regulate the activity of promoters but are not themselves 6 days. Similar results were obtained with transgenic tomotoes for
involved in promoter activity. insects Manduca sexta and Heliothis virescens.

43
l
Continuous exposure to high levels of B. thuringiensis results in l
The antisense RNA technology is used to produce transgenic
development of resistance in some insects, e.g. Plodia interpunctella slow ripening tomato called Flavr savr. The enzyme
and Cadra cautella. There was a > 250 fold increase in resistance of polygalactouronase (PG) degrades pectin, the major component
P. interpunctella after 36 generations and a seven fold increase in of cell wall leading to softening of fruits and deterioration in fruit
C. cautella after 21 generations. This resistance was associated with an quality. Transgenic tomatoes containing antisense construct of
increased susceptibility to Cry IC toxin, i.e. increase in Cry IC binding sites gene encoding PG show reduced expression of PG, and therefore
in the midgut epithelium of these insects. slower ripening and fruit softening.
l
Most of the cases of resistance to Cry proteins may be due to changed
binding properties of midgut receptors.
Applications of Medical Biotechnology
l
The transgenic maize inbreeds and their hybrids were completely
l
A recombinant vaccine contains either a protein or gene
encoding a protein of a pathogenic origin that is immunogenic
resistant to European corn borer Ostrinia nubilalis. Similarly, transgenic
and critical to the pathogen function and produced using
rice plants were significantly resistant to leaf folder and stem borer,
recombinant DNA technology.
while broccoli were completely resistant to diamond black moth.
l
Such vaccines based on recombinant proteins are called
Applications in Food Improvement sub unit vaccines.
l
Biotechnology also promises approach towards improvement of seed l
The genes encoding immunogenic proteins are identified and
storage protein quality, e.g. cereal seed proteins are deficient in lysine isolated from a pathogen and expressed in suitable host for
while those of pulses is deficient in sulphur containing amino acids. mass production of proteins. The proteins are then extracted
l
A new gene encoding a storage protein which is rich in deficient amino and purified and mixed with suitable stabilisers and adjuvants,
acids is introduced into the crop to correct its amino acid deficiency. before being ready for immunisation, e.g. Hepatitis B surface
And the transgene is linked to a seed specific promoter to ensure its antigen (HBsAg) against hepatitis B virus.
expression only in seeds. l
DNA based vaccines are called DNA vaccines, in which the gene
l
Vicillin is the major seed storage protein of pea, it contains 7% lysine encoding the relevant immunogenic protein is isolated, cloned
but no sulphur containing amino acid (methionine and cysteine). and integrated into a suitable expression vector. The
Therefore, its protein is low in sulphur which needs to be ameliorated. preparation is introduced into the individual to be immunised.
For this, the gene SFAS for storage protein of sunflower albumin is The gene ultimately gets expressed in the individual and
extracted and fused with vicilin gene promoter. The promoter is known immunogenic protein produced invokes both humoral and
to confer on genes that correct developmental and tissue specific cell-mediated immunities.
expression and accumulation of proteins in seeds. l
Immuno-PCR technique uses PCR amplification of a marker
l
Seed storage protein quality is based on isolation and modification of DNA segment attached to an antibody for detection of antigen
concerned protein encoding gene sequence, either by replacing one or for which this antibody is specific. The PCR products are
few codons with the selected codons or by inserting one or few selected analysed by gel electrophoresis or labelled using fluorochromes
additional codons at appropriate sites. and haptens. The antigen antibody complex will be formed only
l
Zein, the prolamine storage proteins of cereals are deficient in essential in those wells containing the target antigen.
amino acids lysine and tryptophan. Single lysine replacements in l
A large number of human genes encoding pharmaceutically
N-terminal coding sequence as well as within and between peptide valuable proteins have been cloned and expressed in
repeats and double lysine replacement constructs of prolamine genes microorganisms, particularly E. coli and yeast. Several of the
have been prepared. recombinant proteins are used in treatment of various diseases
l
In another case, the 7S legume seed storage protein, -phaseolin gene such as diabetes (by insulin), dwarfism (by human growth
promoter was transferred to rice. Such transgenic rice plants expressed hormone), cancer (by interferons, interleukins, etc), thromosis
the gene in their endosperm and some showed at least 4% of their total (streptokinase) and AIDS (by interferons, granulocyte
proteins to be -phaseolin. macrophage colony stimulating factor).
l
Rice gt 1 gene encodes the major rice seed storage protein and has been Gene therapy may be classified into two types :
modified to encode higher levels of lysine, tryptophan and methionine l
Germ line gene therapy, in which the sperms or eggs are
and is driven by its own promoter. modified by the introduction of functional genes which
l
Scientists have also synthesised and patented a gene encoding a integrates into their genome. This change due to gene therapy is
protein, called CP 3-5 containing 35% lysine and 22% methionine. The heritable and highly effective in treating genetic disorders but
gene CP 3-5 was coupled with seed specific promoters and transferred not applied in humans due to the various technical and ethical
into maize. reasons.
l
An antisense gene is produced by inverting or reversing the orientation
l
Somatic cell gene therapy, in which the gene is introduced in
of the protein encoding region of a gene in relation to its promoter. As a somatic cells, especially of those tissues where expression of
result, the natural sense strand of the gene becomes oriented in 3' 5' concerned gene is critical for health. This approach is again
direction with reference to its promoter and is transcribed. The RNA divided into two groups on the basis of end result of process.
produced by this gene has the same sequence as the antisense strand of l
Augmentation therapy is the type of somatic cell gene therapy
normal gene (except for T in place of U) and is therefore, known as in which the functional gene is introduced in addition to the
antisense RNA. defective gene endogenous to the cell. i.e. the modified cell
contains both the defective as well as normal copies of gene.

44
l
Another approach involves targetted gene transfer. It uses Applications of Environmental Biotechnology
homologous recombination to replace the endogenous gene l
The use of biological agents or their components to convert relatively
with the functional introduced gene. It may be used either to
diffuse and inconvenient to use sources of energy (e.g. biomass and
inactivate a functional endogenous gene or to correct a
sunlight) into more energy dense and convenient to use fuels such as
defective one.
methane, ethanol, butanol, biodiesel and hydrogen, constitutes
Applications of Industrial Biotechnology Fuel Biotechnology. Such biologically produced fuels are called biofuels.
l
When an organic compound is modified by chemically defined
l
Plant species that are efficient users of solar energy for converting CO2
reactions, catalysed by enzymes present in cells, into a product into biomass which can be used as a source of energy called energy
that is recoverable is called biotransformation. It is performed crops.
by microbes, plants as well animal cells. Both the substrate and l
The biomass may be available in the form of wood (lignocellulose)
the product are not involved in primary or secondary e.g. Butea monosperma, casuarina, Eucalyptus, Melia and Tamarix
metabolism of organism employed. diocia, etc produce firewood.
l
Biotransfromation is routinely used for commercial production l
Starch, e.g. cereals, millets, root, tuber crops produce bioethanol.
of several useful antibiotics. Semisynthetic antibiotics, l
Sugar, e.g. sugarcane and sugarbeet also produce bioethanol.
e.g. semisynthetic penicillin and cephalosporins are produced l
Hydrocarbons, e.g. Euphorbia lathyris, Asclepia, Copaifera, algae, etc.
by chemical modification of penicillin nucleus, i.e.
proudce biodiesel.
6-aminopenicillanic acid.
l
Hydrogen, e.g. algae like Chlamydomonas, bacteria like Clostridium is stored
l
Steroid biotransformation is the most important microbial
as metal hydride and produce hydrogen which is a pollution free fuel.
process yielding pharmaceuticals including adrenocortical
hormones, e.g. corticosterone, cortisone and hydrocortisone,
l
Superbug is a modified strain of oil eating bacteria developed by
other therapeutic derivatives as prednisone, triamcinolone, etc. prof. Anand M. Chakraborty. The process of working of these GMOs
which clean up several contaminants from the environment is called
l
Microbes are employed to recover valuable metals from low
bioremediation.
grade ores and from dilute solutions for which conventional
metallurgical processes are uneconomical. The process is l
Biodegradation is the chemical dissolution of materials by bacteria,
referred to as bio-leaching or microbial ore leaching. fungi or other means. It is different from the process of decomposition
occurring naturally. It is usually done for vast range of organic
l
The process involves bacteria mainly Thiobacillus ferrooxidans
compounds including hydrocarbons, polyaromatic hydrocarbons,
which induces leaching of metal sulphate from low grade ores
antibiotics, xenobiotics, etc.
containing insoluble metal sulphides, e.g. copper or iron, zinc,
etc. It requires extremely low pH for growth and activity and
l
The compounds that resist biodegradation and persist in the
derives energy by oxidising metal sulphides. environment are called recalcitrants.

fjohtu cum dSz 'k dkslZ

AIPMT 2016

45
Molecular Basis of Inheritance
DNA AS GENETIC MATERIAL
Genetic Material : DNA Vs RNA
Studies have revealed that RNA is also the genetic material in some viruses. It has a reactive 2 OH group and is unstable due
to the presence of uracil instead of thymine as in DNA.
It thus, mutates at a faster rate and evolves faster. Such rapid changes are harmful for other life forms. DNA, on the other edge,
is less reactive and stable enough to cause major change, hence is considered better for transmission of genetic information.

DNA : Structure
In 1953, Watson and Crick proposed the double helical structure of DNA, on the basis of observations deduced from X-ray
diffraction crystallographic studies of DNA by Rosalind Franklin.
Together, they were awarded with Nobel Prize in Physiology and Medicine for their remarkable discovery in 1962.
Nitrogenous base
2 nm There are two types of base, i.e. Purine
5 3 Phosphoric acid 5 and Pyrimidine. Purine include
As a component of adenine and guanine, while pyrimidine
nucleotide, it is also PO4 includes thymine and cytosine.
Central axis
involved in
The axis at which
phosphodiester
whole DNA strands
linkage.
revolve around.
CH2
3rd C atom of first
sugar molecule O
3.4 nm
C
Minor
groove O
O P=O
Phosphodiester bond
It is a linkage between two O
sugar and a phosphoric acid Base
0.34 nm is involved in bonding. C H2
(COPOC)
Major O
groove 5th C atom of second
sugar molecule
Sugar phosphate backbone Deoxyribose sugar
OH
Phosphoric acid, i.e. H3PO4 having The five carbon sugar,
3
3 reactive OH groups out of which |
Major 2 are involved in forming backbone. which has C H linkage
axis The both strands are antiparallel |
to each other. at carbon no. 2.
Hydrogen bonds
The nitrogenous bases are held together by hydrogen
bonds. The bonds ultimately held to strands of DNA.
The base G MMMMM C has 3 H-bonds, while base
A::::: T have two hydrogen bonds.

In 1950, Erwin Chargaff formulated important postulates regarding DNA structure, i.e. its base composition, which are
summarised as Chargaffs rules.
A base pair must consist of a purine and pyrimidine because
(i) The space available between two chains of DNA, i.e. 2 nm, can accommodate only such pair.
(ii) There is a perfect match between hydrogen donor and acceptor sites on two bases such that A pairs with T and G pairs
with C.

Packaging of DNA
In prokaryotes, even though there is absence of well organised nucleus, DNA is held with some proteins in nucleoid. In
eukaryotes, the major protein involved in packaging of DNA is histone. A set of positively charged proteins (rich in lysine and
arginine) organises to form histone octamer (eight protein molecules).
The best accepted model, proposed for explaining the packaging of DNA is nucleosome model.

64
It can be understored by following diagrammatic representation.

At the simplest level, chromatin


is a double-stranded helical
structure. DNA double helix

2 nm

DNA is complexed Each nucleosome consists A chromatosome


with histones to of eight histone proteins around consists of a nucleosome
form nucleosomes. which the DNA wraps 1.65 times. plus the H1 histone.

Histone H1

Nucleosome core
of eight histone
These 30 nm fibres molecules
11 nm
form loops averaging Chromatosome
300 nm in length. The nucleosomes
300 nm fold up to produce
a 30 nm fibre.

The 300 nm fibres are


compressed and folded to
250 nm wide fibre
produce a 250 nm wide fibre. 30 nm Tight coiling of the 250 nm
fibre produces the chromatid
of a chromosome.

1400 nm
700 nm

Packaging of DNA at different levels

Comparative Structure of Various Types of DNA


Characters A B C D Z
Handedness Right Right Right Right Left
Base pairs / Turn 11.0 10.0 9.3 8.0 12.0
Helix diameter () 23 19 19 16.7 18
Helix rise per bp 2.92 3.36 3.32 3.03 3.52-4.13
Occurrence in biological world Rare Common Less common No In some cells

DNA Replication
l
DNA replicates semiconservatively, the process is guided by the complementary H-bonding. This biological process occurs
in all living organisms and is the basis for biological inheritance.
l
Semiconservative replication of chromosome was experimentally proved by Taylor (1957) in Vicia faba using triradiated
thymidine. It is also experimentally proved by Meselson and Stahl (1958) by conducting following experiment:
(i) Culturing E. coli for many generation in N15 containing culture medium.
(ii) N15 was incorporated as nitrogenous base, in newly synthesised DNA.
(iii) Heavy DNA separated from normal by centrifugation in CsCl (Cesium chloride) density gradient.
(iv) E. coli cells having N15 was transferred to N14 medium.
(v) Extraction of DNA from E. coli cells in earlier step and density of ds DNA in CsCl centrifugation was measured.
l
After one generation, resultant DNA was hybrid with one strand having N15 isotope and other having N14 .
l
Three modes of DNA replication were assumed.
l
In DNA it was found that replication is of semiconservative type, although it can be thought of to operate in conservative or
dispersive mode too.

65
Following are the different enzymes with its
Process of DNA Replication function in DNA replication:
The process of DNA replication occur in nucleus of the cell. It begins at the
origin of replication with a nick or incision made by endonuclease on one Enzymes for DNA replication and their
strand of DNA. Further opening/unzipping of more nucleotides takes place functions
by helicase. It leads to the formation of replication fork. DNA replication
Enzymes Functions
occurs in 5 3 direction. It is continuous on one strand called leading
DNA helicase The enzyme that is responsible for the
strand and in form of small fragments, by forming loop (trombone loop) at unwinding of the double helix by disrupting
another strand (lagging strand). the hydrogen bonds between the
antiparallel strands.
Leading strand Origin of replication DNA gyrase An enzyme that relieves any form of
tension thats produced after the DNA
double helix is unwound.
SSB (Single The proteins that bind to the outside of the
Lagging strand Stranded two single template strands and prevent
Binding) the separated strands of DNA from
proteins re-anneling.
Primase An enzyme that builds RNA primers and
assembles them at the origin of replication
Okazaki Fragments site.
Small 200 bp segments DNA An enzyme that creates complementary
3 5 synthesised by strands of DNA during replication.
polymerase-III
5 3 DNA polymerase-III at Attaches to RNA primer, can only add
lagging strand. nucleotides to a 3-end of a DNA strand.
Single Stranded
Template DNA The enzyme that removes RNA primers
Binding Protein Helicases
DNA polymerase-I and replaces them with the appropriate
The protein complex The enzyme which unwind the
DNA nucleotides.
which maintain the double helical DNA for its replication.
DNA ligase The enzyme that joins DNA fragments
single stranded DNA,
together by phosphodiester bonds.
It prevent the Primase
recoling of DNA. The enzyme which synthesise
Note RNA primer is composed of multiple bases that
the RNA primer.
DNA Polymerase It is removed by DNA get attached (through hydrogen bonds) to the
The enzyme DNA- polymerase-I, before DNA- template strand to initiate the DNA replication.
polymerase was ligase joins these fragments.
discovered by
Kornberg in 1957.
The DNA
3
3 5 Lagging/Discontinuous Strand
RNA
polymerase-III (Trombone Loop) The other nucleic acid present in cell is RNA,
5
catalyze DNA As this strand is already 5 3,
replication in i.e. ribonucleic acid. It is present predominantly
Priming site the parallel synthesis cannot
prokaryotes, in in cytoplasm and mostly in the form of single
RNA Primer take place (DNA
eukaryotes it is replication always takes place in strand. The pyrimidine, i.e. thymine of DNA is
done by DNA These are small (10-60 bp) RNA
fragments, synthesised by 53 direction) replaced by uracil in RNA. All normal RNA
polymerase and chains begin with adenine or guanine. The RNA
primase, acts as receptor for
DNA polymerase .
primary nucleotides. can be of following types
Leading/Continuous Strand RNA Types Functions
The strand with the direction 3 5,
mRNA (messenger Carries information specifying
on which the continuous synthesis
RNA) amino acid sequences of proteins
of new strand takes place in 5 3 direction.
from DNA to ribosomes.
Machinery of DNA replication showing the trombone loop tRNA (transfer RNA) Transfer amino acids to the protein
synthesis machinery to produce
proteins.
rRNA (ribosomal It is a part of protein synthesis
Sense and Antisense Strands RNA) machinery. Plays structural and
Both the strands of DNA do not take part in controlling heredity and metabolism. Only catalytic roles in ribosomes.
one of them does so. The DNA strand which function as template for RNA synthesis is si RNA (small Used for gene regulation.
known as template strand or minus () strand or antisense strand or non-coding interfering RNA)
strand. Its complementary strand is named non-template strand or plus ( + ) strand or sn RNA (small Used to edit mRNA, regulate gene
sense strand or coding strand. nuclear RNA) expression and maintain
chromosome tips (telomeres).

66
There are three types of rRNA present
Most Primitive Hereditary Vectors-RNA (i) High molecular weight rRNA (mol. wt > 1
Experiments in the 1960s, showed that messenger RNA has the ability to store genetic million)
information, while transfer and ribosomal RNA have the ability to translate genetic e.g. 21 S 29 S rRNA.
information to proteins. Experiments performed two decades later showed that some RNAs (ii) High molecular weight rRNA (mol. wt < 1
can even act as enzyme to self edit their own genetic code. These results raised questions million)
like why RNA play so many roles in the flow of genetic information? and why genetic e.g. 12 S 18 S rRNA.
information is stored in DNA, if RNA alone could do the job? RNA has great capability as
(iii) Low molecular weight rRNA (mol. wt ~
genetic molecule, if one had to carry on hereditary processes on its own.
40000)
It now seems certain that RNA was the first molecule of heredity, so it evolved all the e.g. 5 S rRNA.
essential methods for storing and expressing genetic information before DNA came into the
scene. However, single stranded RNA is rather unstable and is easily damaged by enzyme.
By essentially double the existing RNA molecule and using deoxyribose sugar instead of Transfer RNA or tRNA or
ribose, DNA evolved as a much more stable form to pass genetic information with accuracy. Soluble RNA
It makes about 10 - 20% of total cellular RNA
Messenger RNA or mRNA or Template RNA with sedimentation coefficient of 3.8 S. It
contain 73 - 93 nucleotides.
It makes 3-5% of total cellular RNA. The sedimentation coefficient of mRNA
is 8 S. The name messenger RNA was proposed by Jacob and Monod (1961). tRNA is synthesised in nucleus on DNA
templates. About 0.25% of DNA codes for
The structural components of mRNA includes tRNA. The chief function of tRNA is to carry
(i) CAP (at 5 end) (ii) Non-coding region-1 amino acids to ribosomes for protein
(iii) Initiation codon (AUG) (iv) Coding region synthesis.
(v) Termination codon (vi) Non-coding region-2
(vii) Poly A sequence (at 3 end) Amino
AUG UAA or UAG or UGA acid
(Initiation codon) (Termination codon) Amino acid
binding site
5 3 ACC
Synthetase site
Cap NC 1 Coding NC 2 Poly (A)
(10-100) region (50-150) sequences
nucleotides 1600 nucleotides nucleotides (200-250
nucleotides)
Structure of mRNA
The mRNA formed in nucleus, comes out with proteins into cytoplasm and
normally swim as spherical balls, known as informosomes.
Ribosome
Ribosomal RNA or rRNA recognition site

It makes about 80% or more of total cellular RNA. It is the basic constituent of Anticodon site
ribosomes and developed from the Nucleolar Organiser Region (NOR) of
chromosomes in eukaryotes. In prokaryotes, it is developed from rDNA. tRNA (a) The binding sites

Unpaired
bases
TC loop 5

Paired bases 3

Coiled Uncoiled region CCA terminus


regions DHU loop

Anticodon loop

Structure of ribosomal RNA (schematic) (b) The tertiary structure

67
(iii) To improve tools for data analysis.
TATA Box (iv) To transfer the technologies to other sectors, such
The TATA box of the DNA is the site of assembly of a preinitiation as industries.
complex that contains the general transcription factors and the polymerase. (v) To address the ethical, legal and social
implications of the results obtained from the
(a) Chicken project.
ovalbumin GAGGCTATATATTCCCCAGGGCTCAGCCAGTGTCTGTACA (vi) To identify thousands of genetic markers and map
Rabbit them in the genome.
-globin TTGGGCATAAAAGGCAGAGCAGGGCAGTGCTGCTAACACT
Mouse
-globin GAGCATATAAGGTGAGGTAGGATCAGTTGCTCCTCACATTT Strategy and Methodology
major TBP
NH2 Methodology of HGP includes following three stages:
1. Mapping The physical and genetic maps of
human genome are prepared by using molecular
COOH markers, microsatellites, simple sequence repeats
T A T A A A T A
DNA or sequence tagged sites and PCR amplification of
(c) particular microsatellites.
2. Sequencing The method employed for
(b)
sequencing the entire genomic DNA of human
The TATA box within eukaryotic promotors include
have two basic approach.
(a) Nucleotide sequence of the region just upstream from the site where
transcription is initiated in three different eukaryotic genes. The TATA box is Sequencing
indicated by the grey shading.
(b) Three-dimensional structure of the TATA Binding Protein (TBP) from the Expressed sequence Sequence annotation
flowering plant Arabidopsis thaliana as it sits astride the DNA. tagging method It involves determining the
(c) Binding of TBP is accompanied by a dramatic distortion in the conformation of This method involves complete genome sequence
the DNA helix, which unwinds over an eight base pair stretch. identifying all those genes including all the coding and
that are expressed as RNA. non-coding sequences and
This is represented as ESTs then assigning functions to
Genetic Code is not always Universal: Do you different regions in the sequence.
know? 3. Generation of physical and genetic maps:
In few organisms, some of the codons code uniquely different from rest of the The genetic and physical maps of genome were
organisms (universal). Some of them are tabulated below. generated by using information on:
Codons* (i) Polymorphism of restriction endonuclease
AGA recognition sites.
UGA AUA AGG CUN CGG (ii) Some repetitive DNA sequences called
Normal code assignment Stop lle Arg Leu Arg
microsatellites.
Vertebrates Trp Met Stop + +
Yeasts (Saccharomyces Trp Met + Thr + The HGP Strategy
cerevisiae)
Clones isolated from a genomic library were ordered
Filamentous fungi Trp + + + +
into a detailed physical map, then individual clones
Higher plants + + + + Trp
were sequenced by shotgun sequencing protocols.
Chlamydomonas reinhardtii ? + + + ? The strategy used by the commercial sequencing effort
*N indicates any nucleotide; + codon has the same meaning as in the normal eliminated the step of creating the physical map and
code?, codon not observed in this mitochondrial genome.
sequenced the entire genome by shotgun cloning.
Genomic DNA
Human Genome Project (HGP) DNA is digested into fragments; fragments
inserted into Bacterial Artificial Chromosomes (BAC)
HGP is a comprehensive international research effort dedicated to
map the entire genome by determining the sequence of nucleotides Fragments are identified and mapped.
in the DNA of each of the 22 + X and Y-chromosomes and to study
the functions of human genes. It has also been called International BAC to be sequenced is fragmented;
Human Genome Sequencing Consortium. fragments sequenced at random.

Goals of HGP Sequence overlaps reveal final sequence.

(i) To determine the sequence of the 3 billion base pairs present in Final sequence
the human genome.
It clearly showed that only 1.1%-1.4% part of human
(ii) To identify and determine the functions of approximately all
genome code for proteins.
20000-25000 genes in human DNA.

68
Applications of DNA Fingerprinting
DNA Fingerprinting
DNA fingerprinting is the method of identifying DNAs (i) Forensic uses In criminal investigations, DNA from samples of
of different persons by locating differences in the hair, body fluids or skin at a crime scene is compared with
arrangement of nucleotides in those specific regions in those obtained from suspected perpetrators. DNA typing is
DNA sequence, which are repeated several times. This also used to identify the remains of unknown individuals, as in
technique was invented in 1977 by a British geneticist, the identification of the soldiers or to identify the
Sir Alec Jeffreys at Leicester University. unrecognisable bodies of people killed in political violence or
accidents whose bodies are
Procedure of DNA Fingerprinting (ii) Paternity Paternity determination is possible with DNA typing
because half of the fathers DNA is contained in the childs
Step I. Isolation of DNA DNA must be recovered genetic material.
from the cells or tissues of the body. Only a (iii) Anthropology DNA fingerprinting has also been used
small amount of tissue like blood, saliva, extensively to identify human remains, solving long-standing
semen, hair or skin is required as source. mysteries. DNA has had an enormous impact on anthropology
Step II. Cutting, sizing and sorting Special enzymes as well.
called restriction enzymes are used to cut the (iv) Wildlife management The more the genetic makeup of
DNA at specific places. The DNA pieces are natural populations is understood, the better conservation and
sorted according to size by a sieving management plans will be executed.
technique called electrophoresis. The DNA
pieces are passed through a gel made from
seaweed agarose. Mobile Genetic Elements of Genome
Step III. Transfer of DNA to nylon The distribution of Transposons are special small fragments of DNA (700 to 40,000 base pairs
DNA pieces is transferred to a nylon sheet by long) that can move from one region of DNA molecule to another. These are
placing the sheet on the gel and soaking them also called jumping genes. These jumping genes were discovered by Barbara
overnight. The process being called as McClintock (1950s) in corn. These occur in all organisms and have been
southern hybridisation. studied most thoroughly in microorganisms. All transposons contain the
information for their own transposition. For example, the simplest transposons
Step IV. Probing Adding radioactive or coloured
(also called insertion sequences) contain only a gene that codes for an enzyme
probes to the nylon sheet produces a pattern
transposase (catalyses the cutting and resealing of DNA during
called the DNA fingerprint. Each probe transposition) along with recognition sites (short, inverted, repeat sequences
typically sticks in only one or two specific of DNA for enzyme recognition between transposons and rest DNA).
places on the nylon sheet.
Transposons with antibiotic resistance genes are of practical interest, but
Step V. DNA fingerprint The final DNA fingerprint is there is no limitation on the kinds of genes which transposons can have. Now,
built by using several probes (5-10 or more) we can say that transposons provide a natural mechanism for movement of
simultaneously. The segments marked with genes from one chromosome to another and as these may be carried between
probes are exposed on X-ray film, where they cells or plasmids or viruses they can also spread from one organism to another
form a characteristic pattern of black bars the or from one species to another. Thus, today transposons are considered as a
DNA fingerprint. potentially powerful mediator of evolution in organisms.

CBSE CHAPTERWISE
SOLVED PAPERS (2015-2008)

For
CLASS

XII

69
1. Transformation was discovered by [CBSE AIPMT 2014] 9. If double stranded DNA with N undergoes 15

(a) Meselson and Stahl (b) Hershey and Chase replication twice in a normal medium, then which of
(c) Griffith (d) Watson and Crick the following is correct?
2. Griffiths experiment proved that (a) Half bacteria contain 15 N in DNA
(a) DNA is the genetic material (b) All four contains 15 N in DNA
(b) there are two strains of Streptococcus pneumoniae (c) None contains 15 N in DNA
(c) some transforming factors transformed rough bacteria into (d) 3/4 bacteria 15 N in DNA
smooth bacteria 10. ......stabilises the chain in the single stranded form to
(d) RNA is the transforming factor reduce the energy needed to unwind the DNA helix.
3. Which of the following is not relevant to the structure of (a) Topoisomerase
double helical DNA. (b) Helicase
(a) The helix makes one complete spiral turn every 34 (c) SSB protein
(d) Gyrase
(b) The diameter of the helix is 20
11. Identify the incorrect difference from the following
(c) The distance between adjacent nucleotide is 3.4
given differences between leading and lagging strand
(d) Each strand of helix has a backbone made up of and select the correct option.
alternating ribose sugar and phosphate
Leading strand Lagging stand
4. The result of the following reaction/experiment carried (a) It grows continuously as a It is formed initially as short
out by Avery et al. on Streptococcus pneumoniae has single piece segments called Okazaki
proved conclusively that DNA is the genetic material segments
(a) live R strain + DNA from S strain + DNAse (b) It needs a single RNA primer Each segment needs a
to start its growth separate RNA primer to start
(b) heat killed R strain + DNA from S strain + DNAse
(c) It does not need DNA ligase DNA ligase is needed to join
(c) live R strain + DNA from S strain + RNAse Okazaki fragments
(d) live R strain + Denatured DNA of S strain + protease (d) Direction of growth is 3 5 Direction of growth of each
5. In Hershey and Chase experiments, radioactive P 32 Okazaki fragment is 3 5
was used to culture bacteriophages which resulted in 12. Histones are a set of positively charged proteins which
radioactive [Kerala CEE 2011] are rich in
(a) viral DNA (a) lysine and asparagine (b) lysine and arginine
(b) bacterial capsule (c) valine and glutamine (d) valine and serine
(c) viral proteins
(d) protein capsule of bacteriophage
13. Which of the following is a major difference in
replication in prokaryotes and eukaryotes?
6. Which property among those listed below is not a
(a) Replication in prokaryotes is conservative while it is
criteria for a molecule to act as a genetic material?
semiconservative in eukaryotes
[Kerala CEE 2014]
(b) Prokaryotes do not require small RNA primer for initiation of
(a) Generate its replica replication as in eukaryotes
(b) Chemically and structurally stable
(c) Prokaryotes have single origin of replication and
(c) Mutate slowly to facilitate evolution
eukaryotes have multiple origin of replication
(d) Destroy itself after every cell cycle
(d) All of the above
7. A base pair consists of a purine and pyrimidine
14. Select the correct option [CBSE AIPMT 2014]
because
(a) the space available between two chains of DNA, i.e. 2 nm Direction of RNA synthesis Direction of reading of the
template DNA strand
can accommodate only such pair
(b) there is perfect match between hydrogen donor and (a) 5 3 3 5
(b) 3 5 5 3
acceptor sites on two bases
(c) 5 3 5 3
(c) two polynucleotide chains of DNA are complementary to
(d) 3 5 3 5
each other
(d) All of the above 15. All cells undergo semiconservative mode of
8. Which of the following Chargaff's rule is incorrect? replication, but the mechanism and enzyme differs in
(a) The DNA molecule has equal AT and GC base pairs eukaryotic and prokaryotic cells. Identify the correct
(b) Purines (A + G) are always equal to pyrimidines (T + C) statements regarding replication in prokaryotic cells.
(c) The amount of A is always equal to that of T and the I. DNA ligase joins the RNA primers.
amount of G is always equal to that of C. II. DNA polymerase catalyses polymerisation in 5 3
(d) The base ratio A + T / G + C varies for a given species direction.

70
III. Okazaki fragments form the continuous strand. 22. The number of base pairs per helical turn in Z-DNA is
IV. Leading strand is synthesised in 5 3 direction. (a) 10 (b) 11
(a) I, II and IV (b) Only II (c) 12 (d) 13
(c) II and IV (d) I, II, III and IV 23. Out of AT, GC pairing, bases of DNA may exist in
16. Which option shows correctly labelled region in the alternate valency state owing to a arrangement called
given diagram of DNA replication? [Guj CET 2014] [WB JEE 2009]
5 (a) tautomerisational mutation (b) analogue substitution
A (c) point mutation (d) frameshift mutation
3
24. An analysis of a DNA (double stranded) sample
yielded 18% cytosine. What would be the percentage of
Lagging strand C other bases in the sample? [Haryana PMT 2009]
(a) T32%, A32%, G18% (b) T32%, A18%, G32%
(c) T18%, A32%, G32% (d) T40%, A22%, G20%
5
B
25. Which one of the following pairs of nitrogenous base of
3 nucleic acids, is wrongly matched with the category
(a) A and C (b) Only C (c) A and B (d) B and C mentioned against it? [CBSE AIPMT 2008]
17. Choose the correct statement. [Kerala CEE 2014] (a) Thymine, uracil Pyrimidines
(a) Haploid content of human DNA is 4.6 10 6 bp (b) Uracil, cytosine Pyrimidines
(b) A nitrogenous base is linked to pentose sugar through (c) Guanine, adenine Purines
phosphodiester linkage (d) Adenine, thymine Purines
(c) X-ray diffraction data of Maurice Wilkins and Rosalind 26. During replication of a bacterial chromosome, DNA
Franklin was the basis of Watson and Crick's DNA model synthesis starts from a replication origin site and
(d) DNA, an acidic substance was first identified by Watson
[Manipal 2007]
and Crick
(a) RNA primers are involved
18. Identify the wrong statement about DNA.
(b) is facilitated by telomerase
[Kerala CEE 2014]
(c) moves in one direction of the site
(a) The length of DNA is defined as the number of base pairs (d) moves in bi-directional way
present in it
27. E.Coli about to replicate was placed in a medium
(b) Cytosine is common to both DNA and RNA containing radioactive thymidine for 5 minutes. Then
(c) In a nucleotide, the nitrogenous base is linked to it was made to replicate in a normal medium. Which of
phosphate group. the following observation will be correct? [AIIMS 2007]
(d) Deoxythymidine is a nucleoside
(a) Both the strands of DNA will be radioactive
19. When the DNA replication starts the [AIIMS 2012] (b) One strand radioactive
(a) leading strand produces Okazaki fragment (c) Each half strand radioactive
(b) hydrogen bonds between the nucleotides of two strands (d) None is radioactive
break 28. The strand of DNA acting as template for mRNA
(c) phosphodiester bond between the adjacent nucleotides transcription is
break I. coding strand II. non-coding strand
(d) bonds between the nitrogen base and deoxyribose sugar III. sense strand IV. ant sense strand
break
The correct answer is
20. In prokaryotes, the process of replication is catalysed (a) I and III (b) I and IV
by the following enzymes. Identify, which of the (c) II and IV (d) II and III
enzymes best coordinate with the role [AIIMS 2013] 29. Select the incorrect statement from the following
(a) HelicaseJoins the ends of DNA segments I. Each gene contains a specific promoter region and
(b) DNA polymerase-ISynthesises DNA a leader sequence for guiding the beginning of
(c) DNA polymerase-IIErases primer and fills gaps transcription.
(d) PrimaseSynthesises RNA primers II. Only one strand of DNA called template strand is
21. Given below is a sample of portion of DNA strand copied by RNA polymerase this strand runs in
giving the base sequence on the opposite strands. 3 5 direction.
What is so special shown in it? [CBSE AIPMT 2011] III. RNA polymerase adds complementary nucleotides
5 GAATTC3, 3 CTTAAG5 forming single stranded mRNA in 3 5 direction.
(a) Deletion mutation IV. Section of DNA that has been transcribed is
(b) Start codon at the 5 end rewound into its original configuration.
(c) Palindromic sequence of base pairs (a) I and II (b) II and IV
(d) Replication completed (c) III and IV (d) None of these

71
30. Read the following statements and choose the correct 38. Which one of the following is not a part of transcription
option. [Kerala CEE 2014] unit in DNA? [CBSE AIPMT 2012]
I. RNA polymerase associates transiently with Rho (a) The inducer (b) A terminator
factor to initiate transcription. (c) A promoter (d) The structural gene
II. In bacteria, transcription and translation takes 39. Which of the following is a correctly described
place in the same compartment. characteristic of genetic code?
III. RNA polymerase-I is responsible for transcription (a) Degenerate One amino acid has more than one triplet
of tRNA. code
IV. When hnRNA undergoes capping process, (b) Universality Given codon specifies same amino acid in
adenylate residues are added at 3 -end in a all organisms
template independent manner. (c) Non-ambiguous Each codon codes for only one amino
V. hnRNA is the precursor of mRNA. acid
(a) Only II (b) II, III and IV (c) III and IV (d) I and V (d) All of the above
31. The diagram shows an important concept in the 40. Choose the wrong statement in the process of protein
genetic implication of DNA. Fill in the blanks A to C.
synthesis.
[NEET 2013]
(a) After uncoiling of DNA molecule, one strand acts as a
A
DNA B
mRNA Protein Proposed
by

C template for the formation of mRNA
(a) Atranscription, Breplication, CJames Watson
(b) In the presence of DNA polymerase enzyme, the mRNA is
(b) Atranslation, Btranscription, CErwin Chargaff
formed based on the triplet codes.
(c) Atranscription, Btranslation, CFrancis Crick
(c) The mRNA that leaves nucleus reaches cytoplasm and
(d) Atranslation, Bextension, CRosalind Franklin
gets attached with 30 S ribosomal subunit
32. If a segment of an mRNA molecule has the sequence (d) The amino acids are transferred from the intracellular
5 GUACCGAUCG 3 , which of the following could have
amino acid pool to the active ribosomes by rRNA.
been the template DNA molecule? [AIIMS 2013]
41. Match the following columns
(a) 5 -GCUAGCCAUG-3 (b) 5 -GUACCGAUGG-3
(c) 5 -CATGGCTAGC-3 (d) 5'-CGATCGGTAC-3 Column I Column II
33. Which of the following recognises the start signal on A. UUU 1. Leu
B. UAG 2. Asp
DNA to initiate transcription C. AUG 3. Stop
(a) TATA box (b) Sigma factor D. GAU 4. Met
(c) Core enzyme (d) Operation E. CUG 5. Phe
34. After transcription, a primary transcript of RNA Codes
complementary to DNA is released. What would be the A B C D E A B C D E
most likely fate of this transcript before being used for (a) 1 3 2 4 5 (b) 1 2 3 4 5
translation and coding of proteins. (c) 5 4 3 2 1 (d) 5 3 4 2 1
I. Removal of exons 42. In a given segment of mRNA undergoing translation
II. Methylguanosine cap at 5 end and the polypeptide synthesised, identify the correct
III. Poly A tail at 3 end codes for given amino acids phenylalanine and serine
IV. Methylguanosine cap at 5 end and sequence of DNA from which this mRNA is
V. Removal of introns. transcribed
(a) I, III and IV (b) II, IV and V (c) II, III and V (d) IV and V I. Sequence of mRNA AUG UUU AUG CCU GUU
35. Select the correct statements regarding mRNA stability UCU UAA
I. Prokaryotic mRNA have a half-life of only a few MetPheMetProValSer
minutes. II. Polypeptide synthesised
II. Regulation of mRNA stability is a way of regulating (a) TACAAATACGGACAAAAGAATT, UAA and UUA
gene expression. (b) TTGAAAUCAGGACAAAGAAUU, CAG and AUU
III. Poly A tails stabilise eukaryotic mRNAs. (c) ATGTTTATGCCTGTTTCTTAA UUU and UCU
IV. Histone mRNA have especially long poly A tails (d) TACAAATACGGACAAAAGAATT, UUU and UCU
(a) I and IV (b) I, II and III (c) Only IV (d) II and III 43. In mRNA, AUG is the initiation codon and UAA, UAG
36. In eukaryote, RNA polymerase-II transcribe and UGA are termination codons, therefore
(a) hnRNA (b) 18 S rRNA polypeptide cannot be synthesised beyond any of these
(c) 28 S rRNA (d) snRNA triplets to the end of mRNA. Then which one of the
37. If one strand of DNA has the nitrogenous base following mRNA can be translated completely.
sequence as ATCTG, what would be the (a) AUG UUC UCC UGG UAA UAU
complementary RNA strand sequence? (b) AUG UUC UCC UGA UGC UAU
[CBSE AIPMT 2012] (c) AUC ACG UAU UUC UGA CUC
(a) TTAGU (b) UAGAC (c) AACTG (d) ATCGU (d) AUG UAU UUC UGC CUC UAG

72
44. An eukaryotic cell, post transcription prepares itself for 52. What would happen if in a gene encoding a
synthesis of proteins, so as to meet its metabolic needs. polypeptide of 50 amino acids 25th (UAC) is mutated to
The modified mRNA translates into proteins through a UAA? [AIIMS 2007]
sequential stepwise process. Identify the correct (a) A polypeptide of 49 amino acids will be formed
sequence of steps in translation and select the correct (b) A polypeptide of 25 amino acids will be formed
option. (c) A polypeptide of 24 amino acids will be formed
I. Amino acylation of tRNA. (d) Two polypeptides of 24 and 25 amino acids will be formed
II. Attachment of larger subunit of ribosome to mRNA- 53. The wild type E. coli cells are growing in normal
tRNA met complex. medium with glucose. They are transferred to a
III. Linking of adjacent amino acids to form a medium containing only lactose as sugar. Which of the
polypeptide. following changes take place?
IV. Codon anticodon reaction between RNA and (a) E. coli cells stop dividing (b) The lac operon is induced
amino acyl tRNA complex. (c) The lac operon is repressed (d) All operons are induced
V. Attachment of mRNA with smaller subunit of 54. Identify the terms for the following given statements
ribosome. and select the correct option.
(a) I II V IV III (b) III II I IV V I. A nucleotide sequence that enables a gene to be
(c) I V IV II III (d) II IV V I III transcribed.
45. Which triplet codon does not have a tRNA associated II. These gene control the operator gene.
with it? [Kerala CEE 2014] III. A segment of DNA that a repressor binds to.
(a) UAA (b) UUA (c) UUU (d) AUU IV. Genes that are coregulated by operon.
46. 3 -AAA TGC GCG ATA-5 is the sequence of (a) IOperator IIStructural gene
nucleotides on a gene, after transcription the mRNA IIIRegulator IVPromoter
formed against it and the sequence of bases in the (b) IPromoter IIOperator
corresponding binding anticodon will be [AIIMS 2014] IIIStructural gene IVRegulator
(a) 5 -UAU-GUT-CCA-UUU-3 and 3 AUA-CAU-GGU-AAA-5 (c) IPromoter II Regulator
(b) 5 -UUU-ACG-CGC-UAC-3 and 3 AAA-UGC-GCG-AUA-5 III-Operator IVStructural genes
(c) 5-UAU-CGC-GCA-UUU-3 and 3 AUA-GCG-CGU-AAA-5 (d) IRegulator IIPromoter
(d) 5-UUU-ACC-TUG-UAU-3 and 3 AAA-UGG-UAC-AUA-5 IIIStructural gene IVOperator
47. Which one of the following statement is not correct 55. In lac operon, if mutation occurs in the middle gene of
the structural gene then [Guj. CET 2014]
during protein synthesis? [KCET 2013]
(a) -galactosidase will not be synthesised
(a) UAA codon codes for lysine
(b) permease will not be synthesised
(b) UGG codon codes for tryptophan
(c) transacetylase will not be synthesised
(c) Cysteine is coded by UGU and UAC codons
(d) lactose digestion will be rapid
(d) Tyrosine is coded by UAU and UAC codons
48. Gene of bacteria and virus code for more than one 56. Select the two correct statements out of the four (I-IV)
given below about lac operon. [CBSE AIPMT 2011]
polypeptide are called [OJEE 2011]
(a) overlapping gene (b) monocistronic gene I. Glucose or galactose may bind with the repressor
(c) polycistronic gene (d) non-ambiguous gene and inactivate it.
49. The reaction [DUMT 2011] II. In the absence of lactose the repressor binds with
the operator region.
Amino acid + ATP Aminoacyl AMP + P-P depicts
(a) amino acid assimilation (b) amino acid transformation III. The z-gene code for permease.
(c) amino acid activation (d) amino acid translocation IV. This was elucidated by Francisco Jacob and
50. The one aspect which is not a salient feature of genetic Jacques Monod.
code, is its being [CBSE AIPMET 2010] Which of the following statements given above are
(a) degenerate (b) ambiguous (c) universal (d) specific correct?
51. Match the following columns. (a) II and III (b) I and III (c) II and IV (d) I and II
57. Which of the following events would occur in
Column I Column II
lac-operon of E.coli when the growth medium has high
A. tRNA 1. Linking of amino acids
concentration of lactose? [KCET 2014]
B. mRNA 2. Transfer of genetic information
C. rRNA 3. Nucleolar organising region (a) The structural genes fail to produce polycistronic mRNA
D. PeptidyI transferase 4. Transfer of amino acid from (b) The repressor protein binds to RNA polymerase and
cytoplasm to ribosome prevents translation
Codes (c) The repressor protein attaches to the promoter sequence
A B C D A B C D and represses the operator
(a) 4 2 3 1 (b) 1 4 3 2 (d) The inducer molecule binds to repressor protein and RNA
(c) 1 2 3 4 (d) 1 3 2 4 polymerase binds to promoter sequence

73
Answers
1. (c) 2. (c) 3. (d) 4. (a) 5. (a) 6. (d) 7. (c) 8. (d) 9. (a) 10. (c)
11. (d) 12. (b) 13. (c) 14. (a) 15. (c) 16. (b) 17. (c) 18. (c) 19. (b) 20. (d)
21. (c) 22. (c) 23. (a) 24. (a) 25. (d) 26. (a) 27. (b) 28. (c) 29. (d) 30. (d)
31. (c) 32. (c) 33. (b) 34. (c) 35. (b) 36. (a) 37. (b) 38. (a) 39. (d) 40. (b)
41. (d) 42. (d) 43. (d) 44. (c) 45. (a) 46. (b) 47. (a) 48. (b) 49. (c) 50. (b)
51. (a) 52. (c) 53. (b) 54. (c) 55. (c) 56. (c) 57. (d)

MASTER
1. Which one of the following is wrongly matched? (c) Nucleic acids from viruses are rightly complexed with
[CBSE AIPMT 2014] nucleic acid binding proteins and so, cannot base pair
(a) Transcription Writing information from DNA to t RNA another
(b) Translation Using information in mRNA to make (d) The genome of bacteriophage 174 is single-stranded
protein 6. Which one of the following does not follow the central
(c) Repressor Binds to operator to stop enzyme dogma of molecular biology? [CBSE AIPMT 2010]
protein synthesis (a) Pea (b) Mucor (c) Chlamydomonas (d) HIV
(d) Operon Structural genes, operator and promoter 7. hnRNA undergoes two additional processing. Out of
2. Find the wrongly matched pair. [Kerala CEE 2014] which, in one of them an unusual nucleotide (methyl
(a) Har Govind Khorana Synthesised RNA guanosine triphosphate) is added to the 5 end of
molecules chemically hnRNA. This is known as [AIIMS 2009]
(b) George Gamow Codon is triplet (a) capping (b) tailing (c) splicing (d) termination
(c) Meselson and StahI Regulation of gene expression 8. Whose experiments cracked the DNA and discovered
(d) Alec Jeffreys DNA fingerprinting unequivocally that genetic code is triplet?
[CBSE AIPMT 2009]
3. A geneticist isolates a gene for a specific trait under
study. And also the corresponding mRNA. She found (a) Nirenberg and Matthaei (b) Hershey and Chase
that the mRNA contains 1000 fewer bases than the (c) Morgan and Chase (d) Beadle and Tatum
DNA sequence. Did she isolate the wrong DNA? 9. The length of DNA molecule greatly exceeds the
(a) Yes, mRNA is made from DNA template and should be the dimensions of the nucleus in eukaryotic cells. How is
same length as DNA sequence. this DNA accommodated? [CBSE AIPMT]
(b) Yes, the mRNA should contain more bases than the DNA (a) Deletion of non-essential genes
sequence because bases flanking the gene are also (b) Super coiling in nucleosomes
transcribed. (c) DNAase digestion
(c) No, the final mRNA contains only exons, the introns were (d) Through elimination of repetitive DNA
removed. 10. Automated DNA sequencers, work on the principle of
(d) No, the mRNA was partially degraded after it was the method developed by
transcribed. (a) Erwin Chargaff (b) Maurice Wilkins
4. The newly synthesised mRNA undergoes few (c) Frederick Sanger (d) Alec Jeffreys
modifications before undergoing translation. In the 11. During transcription RNA polymerase holoenzyme
absence of which of the changes it is unable to undergo binds to a gene promoter and assumes a saddle-like
translation structure. What is its DNA-binding sequence?
(a) 5 guanosine cap (b) poly A tail (a) TTAA (b) AATT (c) CACC (d) TATA
(c) snRNPs (d) None of these 12. The haploid content of human DNA is
5. The DNA from the bacteriophage 174 has a base (a) 3.3 10 6 bp (b) 3.3 10 9 bp
composition of 25% A. 33% T, 24% G and 18% C, which (c) 4.6 10 6 bp (d) 6.6 10 9 bp
of the following best explains this observation? 13. Alec Jeffreys developed the DNA fingerprinting
(a) In viral genomes, the base pairing does not follow the technique. The probe he used was
standard Watson and Crick rules and allow G-A and C-T (a) ribozyme (b) sex chromosomes
base pairs (c) SNP (d) VNTR
(b) Viral genomes are linear and tolerate base pair mismatches.

Answers
1. (a) 2. (c) 3. (c) 4. (c) 5. (d) 6. (d) 7. (a) 8. (a) 9. (b) 10. (c)
11. (d) 12. (b) 13. (d)

74
Mukta Gigras

Amazing facts with proper explanation.

DIGESTIVE SYSTEM
Gut also have taste receptors and can sense the in the formidable environment of stomach. This remarkable
basic tastes in a manner similar to tongue. organism Helicobacter pylori has evolved a set of defenses to
Do you know? escape detection and protection. But this bacteria is not a silent
The gastrointestinal tract has ability to sense and respond coloniser in stomach. It exploits the sea of urea as protection
specifically and differentially to the composition of a meal.The against acidic environment of stomach. A large amount of urea is
fats and proteins do not stimulate the same endocrine and released as a result of protein digestion in stomach and is a
exocrine responses as a meal of pure carbohydrate. The nitrogenous breakdown product of amino acids.
traditional sensory receptors such as osmoreceptors and The outer membrane of H. pylori consists of ureases, an enzyme
stretch receptors did not seem to respond to biomolecules. that convert urea into CO 2 and ammonia. Ammonia being a base
However, new researches and studies have revealed the combines with H+ to create NH+4 ions and reduce acidity. Thus,
existence and role of chemosensory cells or taste receptors. protecting themselves from being killed by acidic environment of
The gastrointestinal tract is the key interface between the stomach. It attacks the lining of stomach which protects it from
food and human body and can sense the basic tastes as the acidic environment, thus causing sores or ulcers. They also
tongue. Its epithelial cells use or express G-protein - coupled interfere with immune system so as to ensure protection. Most
receptors and taste linked G-protein gustducin as taste buds. peptic ulcers are found in duodenum and some in stomach itself.
These receptors taste the luminal content and transmit Though earlier, before the discovery of H. pylori stress, food
signals that regulate nutrient transporter expression and habits and high levels of acid and enzyme production was
nutrient uptake as well as the release of gut hormones and believed to cause peptic ulcers.
neurotransmitters involved in regulation of energy and
Defecation is a complex process taking place by
glucose homeostasis.
combination of two reflexes and voluntary effort.
These receptors play a prominent role in communication Do you know?
between the lumen, epithelium, smooth muscle cells, afferent
Most of the time, the rectum is empty of faeces, as a weak
nerve fibres and brain to trigger adaptive responses that
functional sphincter exists about 20 cm from the anus, at the
affect gastrointestinal function, food intake etc.
juncture between the sigmoid colon and rectum which
Small intestine is the major site where dietary sugars are contributes additional resistance to filling of the rectum. When a
absorbed into the body to provide energy and maintain mass movement forces faeces into the rectum, the desire for
normal metabolism. If glucose gets absorbed in excess, it defecation occurs immediately, including reflex contraction of
may result in obesity. T1R3 and gustducin receptors get rectum and relaxation of anal sphincters. Usually defecation is
expressed in specialised taste cells of gut which sense the initiated by defecation reflexes. One of these is an intrinsic reflex,
presence of glucose within the intestine and promotes mediated by local enteric nervous system in rectal wall. When
secretion of Glucagon Like Peptide 1 (GLP-1). It in turn faeces enter the rectum, distention of the rectal wall initiates
promotes insulin secretion and regulates appetite. afferent signals that spread through the myenteric plex to initiate
Do you know that stomach provides the most peristaltic waves in descending colon, sigmoid and rectum
hostile environment on Earth by being acidic forcing faeces towards the anus. As the peristaltic wave
enough to digest iron nails and yet it allows life approaches the anus, the internal anal sphincter is relaxed by
to sustain? inhibitory signals from myenteric plexus. And if external anal
sphincter is also voluntarily relaxed at the same time, defecation
Human stomach is one of the most hostile environments on
occurs. But this intrinsic myenteric defecation reflex functioning
Earth, since, it bathes in very strong acids and its walls are
by itself is relatively weak, therefore another defecation reflex,
protected by sticky mucus and keeps churning constantly.
i.e. parasympathetic defecation reflex also comes into action for
This makes the stomach uninhabitable by any life form.
effective defecation. This reflex involves sacral segments of
However, it has been discovered that a hardy bacteria thrives
spinal cord. When the nerve endings in the rectum are

88
stimulated, signals are transmitted first into the spinal cord and physical symptoms intensify i.e. cause a continuing tetanic
then back to descending colon, sigmoid, rectum and anus contraction lasting for 2-3 min. Hunger and fullness is regulated
through parasympathetic nerve fibres in the pelvic nerves. by hypothalamus in brain. Once you have fed your body
These signals greatly intensify the peristaltic waves and relax enough food to satisfy its needs, signals registering fullness or
the internal anal sphincter, thus converting the weak effort of satiety are sent to hypothalamus.
intrinsic myenteric defecation reflex into a powerful process of It takes approximately 20 min for fullness signals to transmit
defecation. Such that sometimes it is effective in emptying the from the stomach back to the brain. But if one eats too fast and
large bowel all the way from splenic flexure of colon to anus. doesnt pay attention, it is easy to overside this system and end
Defecation signals also initiate other effects such as taking up eating more than what is required by our body.
deep breath, closure of glottis and contraction of abdominal Hunger contractions are most intense in young, healthy people
wall muscles to force the contents of the colon downward. having high degrees of gastrointestinal tonus. It also increases
However, the defecation reflexes can purposely be activated by by lower than normal levels of blood sugar.
taking a deep breath to move the diaphragm downward and
then contracting the abdominal muscles to increase the Vomiting is a protective reflex in response to
pressure in abdomen, thus forcing fecal contents into rectum to obstruction in any part of gastrointestinal tract.
cause new reflexes. Do you know?
Reflexes initiated in this way are not as effective as those that Vomiting is the process by which the upper gastrointestinal
arise naturally. So, the people who often inhibit their natural tract rids itself of its contents when almost any part of upper
reflexes are likely to become severely constipated. tract becomes excessively irritated, over distended or even
overexcitable. Excess distention or irritation of the duodenum
Flatulence and burping is a sign of normal healthy provides an especially strong stimulus for vomiting. It is a
individual with friendly gut flora. Do you know? protective reflex that removes toxic materials from GI tract before
Everyone has gas and burping and passing gas are absolutely they can be absorbed. However, excessive or prolonged
normal. However, many a people think that they burp or pass vomiting with its loss of gastric acid cause metabolic alkalosis.
gas too often or possess much gas i.e flatulence. The vomiting reflex is co-ordinated through a vomiting center in
Actually gas is air in the digestive tract, caused by swallowing the medulla. The reflex begins with a stimulation of sensory
and breakdown of certain foods in the large intenstine, which receptors and is often accompanied by nausea. These sensory
leaves the body when we burp through mouth or through anus signals originate mainly from pharynx, oesophagaus, stomach
by passing gas. Usually, when we eat or drink too fast, smoke and upper portions of small intestine. These nerve impulses are
or drink carbonated or fizzy drinks, we tend to swallow more air. transmitted by both vagal and sympathetic afferent nerve fibres
Some of this swallowed air is absorbed in small intestine, of to the vomiting centre in brain stem. From here, motor impulses
which it passes to large intestine and is finally passed through that cause actual vomiting are transmitted from vomiting centre
anus as gas. by way of fifth, seventh, ninth, tenth and twelfth cranial nerves to
Another fact that contributes to gas formation is that all the food upper GI tract, through vagal and sympathetic nerves to lower
we eat is not digested by stomach and small intestine. Those GI tract and through spinal nerves to diaphragm and
undigested carbohydrates and fibre of foods passed to large abdominal muscles.
intestine are broken down by bacteria inhabiting the colon and Minutes before vomiting occurs, antiperistalsis begins to occur
produce CO 2 ,H2 , CH4 or H2S gas which is eventually passed i.e. peristalsis up the digestive tract rather than downward. This
through anus. Most carbohydrate containing foods may may begin as far down in the intestinal tract as the ileum, with
produce gas i.e. beans, broccoli, cauliflower, cabbage, the antiperistaltic wave travelling backwards up the intestine at
mushrooms, apples, peaches, whole wheat, fruit drinks, a rate of 2-3 cm/s. These waves can actually push a large share
packaged foods, bread, milk and milk products etc. It is not of lower small intestine contents to duodenum and stomach
necessary that foods producing or causing gas in one person within 3-5 min. As a result, the duodenum becomes overly
will release or cause the same in other person. A healthy distended, which becomes the exciting factor that initiates the
individual may have 18-21 flatulences per day indicating a actual vomiting act.
healthy gut flora. They flourish by feeding upon unused food in Once the vomiting onsets, strong intrinsic contractions occur in
large intestine and also produce some short chain fatty acids both duodenum and stomach along with partial relaxation of
that promote the growth of other beneficial bacteria. oesophageal-stomach sphincter and the vomiting begins to
How do we know and differentiate between move from stomach to oesophagus. After adequate
stimulation of vomiting centre, a specific vomiting act involving
Hunger and fullness. Do you know?
abdominal muscles takes over and expels the contents to the
Stomach hunger involves a complex interaction between the exterior. Apart from the irritative stimuli of GI tract, vomiting can
digestive system, endocrine system and brain. When our body also be triggered by nervous signals arising in areas of brain
needs refueling, one start feeling tired and weak and also find it i.e. in fourth ventricle called chemoreceptor trigger zone. Any
difficult to concentrate and work. The stomach begins to ache electric stimulation of this area or administration of certain
and rumble, which indicates true stomach hunger. On the drugs directly stimulate this zone and initiate vomiting.
contrary, if we do not feed food when the body requires it, the

89
Section-A 9. Can an unfertilised, apomictic embryo sac give
rise to a diploid embryo? If yes, then how?
1. These figures show the gynoecium of
10. Differentiate between biopiracy and biowar.
(a) Papaver
or
(b) Michelia flowers. Honey collection improves, when beehives are
Write the difference in the structure of their kept in crop-field during flowering season.
ovaries. Explain.
Section-C
11. What initiatives were taken for reducing vehicular
air pollution in Delhi? Has air quality improved in
Delhi ?
or
Nature has a carrying capacity for a species.
Explain the statement with suitable examples.
12. Two heterozygous parents are crossed. If the
A B
two loci of genes in them are linked, what would
2. A person who is allergic to pulses is advised to be the distribution of phenotypic features in
take a capsule of Spirulina daily. Give the reason
F1 -generation for a dihybrid cross?
for the advice.
3. The egg of an animal contains 10 chromosomes, of 13. What is ecological succession? Where and why
which one is X-chromosome. How many autosomes would the rate of succession be faster in a newly
created pond or a forest destroyed by a forest fire ?
would there be in the karyotype of this animal?
14. Why are offsprings of oviparous animals at a
4. Where is good ozone present? Why is it called so?
greater risk as compared to offspring of viviparous
5. Name the two forms of reservoir of carbon, that animals ?
regulate carbon cycle in the ecosystem.
15. Fertilization is not an obligatory event for fruit
Section-B formation in certain plants. Explain.
6. Name the first human like hominid. Mention his 16. Most living organisms cannot survive at
temperature above 45C. How are some microbes
food habit and brain capacity.
able to live in habitats with temperatures
7. How milk production is hormonally regulated. exceeding 100C?
8. What makes the nucleopolyhedrovirus a desirable 17. How does the antiparallel orientation of
biological control agent? DNA strands affects its replication?

90
18. Make a list of any three outbreeding devices that 25. (i) How is oogenesis markedly different from
flowering plants have developed and explain how spermatogenesis with respect to the growth
they help to encourage cross-pollination. till puberty in the humans?
19. Write a note on mutualism with examples. (ii) Draw a sectional view of human ovary and
20. If implementation of better techniques and new label the different follicular stages, ovum and
strategies are required to provide more efficient corpus luteum.
care and assistance to people, then why there is a or
statutory ban on amniocentesis? Write the use of What is organic evolution? Give the evidences of
this technique and give reason to justify its ban. organic evolution.
21. (i) Why are grasshopper and Drosophila said to 26. (i) Explain the narrow utilarian, broadly
show male heterogamety? Explain. utilarian and ethical arguments in favour of
(ii) Explain female heterogamety with the help of conservation of biodiversity.
an example. (ii) How is designation of certain areas as hot
22. Read the following base sequence of a certain DNA spots, a step towards biodiversity
strand and answer the question that follow: conservation? Name any two hot spots in
India.
A A G A A T T C A A or
T T C T T A A G T T
(i) Cancer is one of the most dreaded diseases.
(i) What is called a palindromic sequence in a Explain contact inhibition and metastasis
DNA? with respect to the disease.
(ii) Write the palindromic nucleotide sequence (ii) Name the group of genes that have been
shown in the DNA strand given and mention the identified in normal cells that would lead to
enzyme that will recognise such a sequence. cancer. How do these genes cause cancer?
(iii) State the significance of enzymes that identify (iii) Name any two techniques that are useful in
palindromic nucleotide sequences. detecting cancers of internal organs.
(iv) Why are cancer patients, often given
Section-D -interferon as part of the treatment?
23. During biology class, teacher was telling about
microbes and how they are useful for humans.
Hearing this, Harry got confused as he knew that
microbes are dangerous and can only cause
diseases. He asked his teacher about it. The
teacher appreciated him for his question and
WBJEE will be on May 17, 2016
explained the whole class. West Bengal Joint Entrance Examinations Board
(i) What are microbes? (WBJEE Board) has released information
(ii) How are microbes helpful to humans? Explain. brochure of WBJEEM 2016 on December 24, 2016.
According to the WEBJEEM 2016 brochure,
(iii) Name one microbe that is used as the Application form for WBJEEM 2016 will be
(a) household products available from January 5, 2016 onwards. WBJEEM
(b) biofertiliser 2016 will be conducted by the board on May 17,
(iv) Identify the values shown by teacher. 2016 for admission to undergraduate level
professional courses in Engineering, Technology,
Section-E Architecture and Pharmacy in government
colleges and self-financed institutes in West
24. How do Pleiotropy, incomplete dominance, Bengal.The information brochure contains
codominance and polygenic inheritance deviate important details related to the entrance exam
from the observation made by Mendel? Explain like Question Pattern, Ranking and tie breaking,
with the help of example for each. Eligibility criteria for candidates, Availability of
or seats, Application Procedure, Important Dates,
Admit Card Counselling and Admission,
Any recombinant DNA with desired gene is
Syllabus etc.
required in billion copies for commercial use. How
is the amplification done? Explain.

91
Section A Section-C
1. Name the Indian breed of cow that was 11. What is asexual reproduction? Why do algae and fungi
developed through artificial selection and shift to sexual mode of reproduction just before the onset
domestication of wild cows. of adverse conditions?
2. If the length of DNA of E.coli is 1.36 mm, 12. Draw a diagram of human sperm. Label only those parts
calculate the number of base pairs in it. along with their functions that assist the sperm to reach
3. Coelacanth was caught in 1938 in South Africa. and gain entry into the female gamete.
Why is it very significant in the evolutionary 13. What are the attributes that population possess but not
history of vertebrates? an individual?
4. If a patient is advised anti-retroviral therapy, 14. Write critical notes on ground water depletion and ways
which infection is he suffering from? Name the of its replenishment.
causative organism. 15. Do you support Dope test being conducted on sports
5. A multinational company outside India tried to persons participating in a prestigions athletic meet.
sell new varieties of turmeric without proper Give three reasons in support of your answer.
patent rights. What is such an act referred to? Or
How is innate immunity different from the immunity
Section-B that you require through vaccine? Describe any two
6. In GIFT, gametes are transferred to the Fallopian ways by which innate immunity can be a accomplished?
tube. Can gametes be transferred to the uterus to 16. Why plants obtained through micropropagation are
achieve the same result? Explain. termed as somaclones? Name three food plants
7. Not all characters show true dominance. What produced on commercial scale using this method.
are the two other possible types of dominance? 17. Write about the types and roles of antibodies in our body.
Give an example of each.
18. What are ethics and bioethics?
8. Enlist the characters that breeders have tried to How are developed nations taking advantage of
incorporate into crop plants. biopatent laws to exploit the resources of developing
9. Define broad spectrum antibiotics. Give two countries?
examples. 19. Can you explain how Australian marsupial shows
10. What are primary and secondary carnivores? evidences for convergent evolution as well as adaptive
Mention their trophic levels in a food chain. radiation?
Or 20. The length of a DNA molecule in a typical mammalian
Explain the function of reservoir in a nutrient cell is calculated to be approximately 2.2 m. How is the
cycle. List the two types of nutrient cycles in packaging of this long molecule done to accommodate it
nature. within the nucleus of the cell?

92
21. How do organisms cope with stressful external (i) What is the minimum number of pollen grains
environmental conditions which are localised or of that must have been involved in the pollination of
its pistil?
short duration?
(ii) What would have been the mininum number of
22. What is the equation of species area relationship? ovules present in the ovary?
Also write down the significance of the slope of (iii) How many megaspore mother cells were
regression in a species-area relationship? involved?
(iv) What is the minimum number of microspore
Section-D mother cells involved in the above case?
(v) How many male gametes were involved in this
23. Prateeks mother wondered why the mosquitoes case?
were not responding to the mosquito repellant,
Or
which she had been using for the last several years.
Give a schematic representation of oogenesis in
Raj suggested Prateeks mother to change the brand
humans. Mention the number of chromosomes at
of repellant she was using. It worked and was
each stage. Correlate the life phases of individual
effective, to which his mother concluded that the
with the stages of the process.
earlier repellent might have been expired or
degraded over time. Prateek objected to her opinion 25. A pea plant producing yellow coloured and round
and explained to her the reality. seeds is given with unknown genotypes. Explain
(a) What values were reflected by Prateek? how you would find the correct genotypes of the
(b) Why did mosquitoes not respond to the repellant? plant with respect to the traits mentioned.
Explain on the basis of natural selection. Work out the cross and name it.
(c) Cite any two examples of natural selection which Or
you often come across. How did Griffith proved transforming principle in
genetics. Explain the procedure.
Section-E
26. Explain how tolerance to environmental factors
24. A flower of tomato plant following the process of determines distribution of species.
sexual reproduction produces 240 viable seeds.
Or
Answer the following questions giving reasons: What are the various components of biodiversity?
Explain each of them.

Answers of Board Exam Corner (December Issue)


1. Sahiwal 10.
2. 4.6 106 bp. Primary Carnivore Secondary Carnivore
3. Coelacanth has evolved into the first amphibians, they are fish with These are the animals which feed on These are the animals, which feed
stout and strong fins that could move on land and go back to water. the herbivores. on the primary carnivores.
4. He is suffering from AIDS. The virus is HIV. They occupy the third trophic level. They occupy the fourth trophic level.
5. Biopiracy.
Or
6. No, the uterine environment is not congenial for the survival of
The function of reservoir is to meet the deficit that arises due to the
gametes. If directly transferred to the uterus they will undergo
imbalances in the influx and efflux of nutrients.
degeneration or could be phagocytosised and hence, viable zygote
The two types of nutrient cycles are :
would not be formed.
(i) Sedimentary cycle (ii) Gaseous cycles
8. (i) High yield
(ii) Disease and pest resistance
26. Components of biodiversity
(i) Species diversity (ii) Genetic diversity
(iii) Desirable quality
(iii) Ecosystem diversity
(iv) Drought resistance

93
ALFRED RUSSEL
WALLACE ( Jan. 8, 1823- Nov. 7, 1913)
A British humanist, naturalist, geographer, and social critic who
became a public figure in England during the second half of the
19th century, known for his courageous views on scientific, social,
and spiritualistic subjects.

LIFE HISTORY
Alfred Wallace was born in the Welsh village of Llanbadoc, near Usk, Monmouthshire.He was the seventh of nine children born to
Thomas Vere Wallace and Mary Anne Greenell. When Wallace was five years old, his family moved to Hertford where he attended
Hertford Grammar School until financial difficulties forced his family to withdraw him in 1836, when he was aged 14.
Wallace then moved to London to board with his older brother John, a 19 year old apprentice builder. While in London, Alfred
attended lectures and read books at the London Mechanics Institute. He left London in 1837 to live with William and work as his
apprentice for six years. By the end of 1843, Williams business had declined due to difficult economic conditions, and Wallace, at
the age of 20, left London in January.
After a brief period of unemployment, he was hired as a master at the Collegiate School in Leicester to teach drawing,
mapmaking and surveying. Wallace spent many hours at the library in Leicester, he read An Essay on the Principle of Population
by Thomas Malthus, and one evening he met the entomologist Henry Bates. He befriended Wallace and started assesting him in
collecting insects. Wallaces work on the survey involved spending a lot of time outdoors in the countryside, allowing him to
indulge his new passion for collecting insects. During this period, he read avidly, exchanging letters with Bates, Charles Darwins
The Voyage of the Beagle, and Charles Lyells Principles of Geology. Inspired by the chronicles of earlier travelling naturalists,
including Alexander von Humboldt, Charles Darwin and especially William Henry Edwards, Wallace decided that he too wanted
to travel abroad as a naturalist. In 1848, Wallace and Henry Bates left for Brazil. Their intention was to collect insects and other
animal specimens in the Amazon rainforest for their private collections, selling the duplicates to museums and collectors back in
Britain in order to fund the trip.
Wallace continued charting the Rio Negro for four years, collecting specimens that and making notes on the peoples and languages
he encountered, as well as the geography, flora, and fauna. All of the specimens Wallace had on the ship, mostly collected during the
last two and most interesting years of his trip were lost but he managed to save a few notes and pencil sketches.
After his return to the UK, Wallace spent 18 months in London living on the insurance payment. During this period, despite
having lost almost all of the notes from his South American expedition, he wrote six academic papers which included On the
Monkeys of the Amazon and two books; Palm Trees of the Amazon and Their Uses and Travels on the Amazon. He also made
connections with a number of other British naturalistsmost significantly, Darwin. In 1866, Wallace married Annie Mitten. On
7 November, 1913, at the age of 90 years Wallace, died at home in the country house he called Old Orchard, which he had built a
decade earlier.
CONTRIBUTIONS IN BIOLOGY
Wallace had wide-ranging interests from socialism to spiritualism, from island biogeography to life on Mars, from evolution to
land nationalization, stemmed from his profound concern with the moral, social, and political values of human life.
From 1854 to 1862, Wallace travelled through the Malay Archipelago or East Indies to study natural history. His observations
of the marked zoological differences across a narrow strait in the archipelago led to his proposing the zoogeographical
boundary now known as the Wallace line.
Wallace collected more than 126,000 specimens in the Malay Archipelago. One of his better-known species descriptions
during this trip was that of the gliding tree frog Rhacophorus nigropalmatus.
While he was exploring the archipelago, he refined his thoughts about evolution and had his famous insight on natural
selection. In 1858 he sent an article outlining about his theory to Darwin; it was published, along with a description of
Darwins own theory, in the same year. An account of his studies and adventures were eventually published in 1869 as The
Malay Archipelago, which became one of the most popular books of scientific exploration of the 19th century. It was praised
by scientists such as Darwin and Charles Lyell and by non-scientists such as the novelist Joseph Conrad, who called it his
favorite bedside companion and used it as source of information for several of his novels.
While returning from his travels, Wallace organised his collections and gave numerous lectures about his adventures and
discoveries to scientific societies such as the Zoological Society of London.
During the 1860s, Wallace wrote papers and gave lectures defending natural selection. He also corresponded with Darwin
about a variety of topics, including sexual selection,warning colouration, and the possible effect of natural selection on
hybridisation and the divergence of species. In 1865, he began investigating spiritualism.
His formulation of the theory of evolution by natural selection, which he predicted in Charles Darwins published
contributions, is his most outstanding legacy, but it was just one of many controversial issues he studied and wrote about
during his lifetime.
1. Which of the following statements are correct with IV. Salamandra is distinguishable from Bufo on
respect to lipoproteins? the basis of parotid gland.
(a) Molecular mass of lipoproteins is directly proportional to Choose the correct set of statements from given codes.
their density. (a) I, II and IV (b) I and IV
(b) The per cent protein content in lipoprotein increases with (c) II and III (d) l, II, III and IV
molecular mass.
(c) Density of a lipoprotein decreases with increase in protein 5. A few members of a population have reached a
content. favourable habitat with few predatons and
(d) Molecular mass of lipoproteins is inversely proportional to unlimited resources, but their population growth
their density. rate is slower than that of parent population. What
2. The function of calmodulin is to can be possible explanation for this situation?
(a) begin an enzyme cascade by phosphorylating multiple (a) The genetic makeup of these founders may be less
proteins favorable than that of the parent population.
(b) bind with Ca 2+ and regulate the activity of cellular proteins (b) The parent population may still be in an exponential part of
(c) lower blood calcium levels its growth curve and not yet limited by density dependent
(d) serve as a second messenger in a signal transduction factors.
pathways (c) The Allee effect may be operating and there may be not
enough population members present for successful
3. A virus has genome comprising of DNA molecule
reproduction.
with a base composition of A = 24.6%, T = 34.7%,
(d) All of the above
G = 20.2% and C = 20.2%. How would you describe
the DNA and determine whether the molecule is 6. Galactosemia is recessive single gene genetic
linear or circular? disorder, caused due to the mutation in any one of
(a) the DNA is single stranded and if resistant to double the three genes involved in galactose catabolism.
stranded specific nucleases, it is linear. A family consists of 10 normal children with both
(b) the DNA is partially double stranded and if resistant to
parents suffering from galactosemia. It is most
nucleases, it is linear.
(c) the DNA is single stranded and if resistant to double
likely due to
stranded and single strand specific exonucleases, it is (a) reversion (b) epistasis
circular. (c) complementation (d) suppression
(d) None of the above
7. Positional cloning refers to
4. Consider the following statements. (a) using a selection procedure to clone c-DNA.
I. Pisces and Amphibia can be distinguished (b) isolating a gene by PCR using primers from another
from each other on the basis of fundamental species.
differences in the lateral line system. (c) isolating a gene from a specific tissue in which it is being
II. Elasmobranchi and Teleostomi can be expressed.
distinguished from each other on the basis of (d) mapping a gene to a chromosomal region and then
endoskeleton. identifying and cloning a genomic copy of the gene from
III. Scoliodon and Torpedo are distinguished on region.
the basis of electric organ through both belong
the classPisces.

95
8. The uptake of nutrient by bacteria (b) Both the seedling root and coleoptiles perceive and
(a) occurs against relatively small concentration gradients in respond to light in same manner.
free living bacteria. (c) a chemical messenger must travel from the base of
(b) often involves transport proteins that are inducible in coleoptile to tip.
response to the presence of transportable molecules in the (d) the light signal is perceived at the tip of the coleoptile but the
medium and the metabolic needs of cells. growth response occurs a few millimeters below the tip.
(c) occurs only by a process involving sugar phosphorylation.
10. Histone acetylation increases transcription of gene
(d) generally does not involve integral membrane proteins with
because
transmembrane alpha helices.
(a) it increases the DNA histone interaction.
9. Charles Darwin and his son Francis experimented (b) the acetyl groups on histones are recognised by RNA
with photo tropism of grass seedlings by placing a polymerase.
metal for blind fold over different parts of seedlings (c) histone acetylation looses the DNA histone complex
coleoptiles. A simplified version of their results is thereby making it more accessible to RNA polymerase.
shown below. Which of the following statements (d) histone acetylation induces DNA bending which is
best explains their results? recognised by RNA polymerase.
(a) The light signal is perceived below the tip and these cells
cause the coleoptiles to grow towards light.

96

Anda mungkin juga menyukai